SU 18 Gleim

Lakukan tugas rumah & ujian kamu dengan baik sekarang menggunakan Quizwiz!

A job-order cost system uses a predetermined factory overhead rate based on expected volume and expected fixed cost. At the end of the year, underapplied overhead might be explained by which of the following situations? Actual VolumeActual Fixed Costs A. Actual Volume Less than expected Actual Fixed Costs Greater than expected B. Actual Volume Greater than expected Actual Fixed Costs Greater than expected C. Actual Volume Greater than expected Actual Fixed Costs Less than expected D. Actual Volume Less than expected Actual Fixed Costs Less than expected

A. Actual Volume Less than expected Actual Fixed Costs Greater than expected Answer (A) is correct. If too little fixed overhead is applied at the predetermined rate (expected fixed cost ÷ expected volume), the result is underapplied overhead (actual factory overhead exceeds overhead applied). If the actual and expected fixed costs are the same, but the actual volume is less than the expected (denominator) volume, overhead will be underapplied. If the actual volume equals expected volume but actual fixed costs exceed the expected (numerator) fixed costs, overhead is likewise underapplied.

In a job cost system, manufacturing overhead is An IndirectCost of JobsA Necessary Elementin Production A. An Indirect Cost of Jobs Yes A Necessary Element in Production Yes B. An Indirect Cost of Jobs Yes A Necessary Element in Production No C. An Indirect Cost of Jobs No A Necessary Element in Production No D. An Indirect Cost of Jobs No A Necessary Element in Production Yes

A. An Indirect Cost of Jobs Yes A Necessary Element in Production Yes Answer (A) is correct. Factory overhead consists of indirect manufacturing costs that cannot be traced to specific units but are necessarily incurred as part of the production process. Examples are depreciation, utilities expense, insurance, and supervisors' salaries. Factory overhead is usually allocated to products based upon the level of activity during the period, e.g., direct labor hours or machine hours.

In process 2, material G is added when a batch is 60% complete. Ending work-in-process units, which are 50% complete, would be included in the computation of equivalent units for Conversion CostsMaterial G A. Conversion Costs Yes Material G No B. Conversion Costs Yes Material G Yes C. Conversion Costs No Material G No D. Conversion Costs No Material G Yes

A. Conversion Costs Yes Material G No Answer (A) is correct. Conversion costs (direct labor and manufacturing overhead) are the costs of transforming direct materials into finished products. If EWIP is 50% complete, it is presumably 50% complete as to conversion costs (all costs other than direct materials). But if material G is added only at the 60% point, no equivalent units of G have been produced. Thus, EWIP is included in the computation of EUP of conversion costs but not material G.

Costs are accumulated by a responsibility center for control purposes when using Job-Order CostingProcess Costing A. Job-Order CostingYes Process CostingYes B. Job-Order CostingNo Process CostingNo C. Job-Order CostingYes Process CostingNo D. Job-Order CostingNo Process CostingYes

A. Job-Order CostingYes Process CostingYes Answer (A) is correct. A responsibility center is a subunit (part or segment) of an organization whose manager is accountable for a specified set of activities. Both job-order costing and process costing may accumulate their costs by responsibility centers.

Boyle, Inc., makes two products, X and Y, that require allocation of indirect manufacturing costs. The following data was compiled by the accountant before making any allocations: Product XProduct YQuantity produced10,00020,000 Direct manufacturing labor hours 15,000 5,000 Setup hours 500 1,500 The total cost of setting up manufacturing processes and equipment is $400,000. The company uses a job-costing system with a single indirect cost rate. Under this system, allocated costs were $300,000 and $100,000 for X and Y, respectively. If an activity-based system is used, what would be the allocated costs for each product? Product XProduct Y A. Product X $100,000 Product Y $300,000 B. Product X $150,000 Product Y $250,000 C. Product X $250,000 Product Y $150,000 D. Product X $200,000 Product Y $200,000

A. Product X $100,000 Product Y $300,000 Answer (A) is correct. In an ABC system, indirect costs are assigned to activities and then allocated to final cost objects. The indirect costs should be assigned to the setup activity and allocated based on the hours it takes to set up each manufacturing process. The cost assigned to Product X is $100,000 [$400,000 × (500 hours ÷ 2,000 hours)], and the cost assigned to Product Y is $300,000 [$400,000 × (1,500 hours ÷ 2,000 hours)].

The following information pertains to Lap Co.'s Palo Division for the month just ended:NumberCost of of UnitsMaterialsBeginning work-in-process15,000$ 5,500Started during the month40,00018,000Units completed42,500Ending work-in-process12,500 All materials are added at the beginning of the process. Using the weighted-average method, the cost per equivalent unit for materials is A.$0.43 B.$0.55 C.$0.45 D.$0.59

A.$0.43Answer (A) is correct. The weighted-average method does not distinguish between work done in the previous period and that done in the current period. Consequently, given that materials are added at the start of the process, the total EUP equal 55,000 (42,500 units completed + 12,500 units in EWIP), the total cost of materials is $23,500, and the cost per EUP for materials is $0.43 (rounded).

Fact Pattern: Zeta Company is preparing its annual profit plan. As part of its analysis of the profitability of individual products, the controller estimates the amount of overhead that should be allocated to the individual product lines from the information given in the next column:WallSpecialtyMirrorsWindowsUnits produced2525Material moves per product line515Direct labor hours per unit200200Budgeted materials handling costs$50,000 Under a costing system that allocates overhead on the basis of direct labor hours, Zeta Company's materials handling costs allocated to one unit of wall mirrors would be A.$1,000 B.$2,000 C.$5,000 D.$500

A.$1,000Answer (A) is correct. If direct labor hours are used as the allocation base, the $50,000 of costs is allocated over 400 hours of direct labor. Multiplying the 25 units of each product times 200 hours results in 5,000 labor hours for each product, or a total of 10,000 hours. Dividing $50,000 by 10,000 hours results in a cost of $5 per direct labor hour. Multiplying 200 hours times $5 results in an allocation of $1,000 of overhead per unit of product.

The following information pertains to Syl Co.: Sales$800,000Variable costs160,000Fixed costs40,000 What is Syl's breakeven point in sales dollars? A.$50,000 B.$160,000 C.$40,000 D.$200,000

A.$50,000Answer (A) is correct. The breakeven point in sales dollars is the fixed costs divided by the contribution margin ratio. Variable costs equal 20% of sales ($160,000 ÷ $800,000). Hence, the contribution margin ratio is 80%, and the breakeven point in dollars is $50,000 ($40,000 FC ÷ 80%).

Felicity Corporation manufactures a specialty line of dresses using a job-order cost system. During January, the following costs were incurred in completing job J-1:Direct materials$27,400Direct labor9,600Administrative costs2,800Selling costs11,200 Overhead was applied at the rate of $50 per direct labor hour, and job J-1 required 400 direct labor hours. If job J-1 resulted in 4,000 good dresses, the cost of goods sold per unit is A.$14.25 B.$17.75 C.$9.25 D.$14.95

A.$14.25Answer (A) is correct. Cost of goods sold is based on the manufacturing costs incurred in production. It does not include selling or general and administrative expenses. Manufacturing costs consist of direct materials ($27,400), direct labor ($9,600), and overhead (400 direct labor hours × $50 per hour = $20,000). The total of these cost elements is $57,000. Dividing the $57,000 of total manufacturing costs by the 4,000 units produced results in a per-unit cost of $14.25.

Regis Company manufactures plugs used in its manufacturing cycle at a cost of $36 per unit that includes $8 of fixed overhead. Regis needs 30,000 of these plugs annually, and Orlan Company has offered to sell these units to Regis at $33 per unit. If Regis decides to purchase the plugs, $60,000 of the annual fixed overhead applied will be eliminated, and the company may be able to rent the facility previously used for manufacturing the plugs. If the plugs are purchased and the facility rented, Regis Company wishes to realize $100,000 in savings annually. To achieve this goal, the minimum annual rent on the facility must be A.$190,000 B.$70,000 C.$40,000 D.$10,000

A.$190,000Answer (A) is correct. If Regis purchases the plugs, Regis will still incur fixed costs per unit of $6 [$8 - ($60,000 ÷ 30,000 units)], however, since these costs are committed (sunk costs), they are not relevant to this decision. Thus, the relevant cost per unit will be $30 ($36 cost - $6 fixed cost per unit). Without regard to rental of idle production capacity, the company will lose $3 per unit ($33 purchase price - $30 relevant cost) by purchasing the plugs. The total annual loss will be $90,000 (30,000 units × $3). Consequently, to achieve the targeted savings, the minimum annual rent must be $190,000 ($90,000 loss from purchasing + $100,000 targeted savings).

Based on potential sales of 500 units per year, a new product has estimated traceable costs of $990,000. What is the target price to obtain a 15% profit margin on sales? A.$2,329 B.$1,980 C.$1,935 D.$2,277

A.$2,329Answer (A) is correct. Costs of the product must be 85% of sales to achieve a 15% profit on sales. Thus, sales must be $1,164,706 ($990,000 ÷ .85). The price per unit is $2,329 ($1,164,706 ÷ 500).

Albany Mining Corporation uses a process costing system for its ore extraction operations. The following information pertains to work-in-process inventories and operations for the month of May: Completion % Units Materials Conversion WIP on May 1 32,000 60% 20% Started in production 200,000 Completed production 184,000 WIP on May 31 48,000 90% 40% Costs for the month were as follows: BWIP Incurred in May Materials $54,560 $ 468,000 Direct labor 20,320 182,880 Manufacturing overhead 15,240 391,160 $90,120 $1,042,040 Under the FIFO method, Albany Mining's cost per equivalent unit for materials is A.$2.25 B.$2.30 C.$2.51 D.$2.06

A.$2.25Answer (A) is correct. Under the FIFO method, EUP are determined based only on work performed during the current period. Thus, units in beginning work-in-process must be excluded.MaterialsUnits transferred out184,000Add: EWIP (48,000 × 90%)43,200Total completed units227,200Minus: BWIP (32,000 × 60%)(19,200)EUP208,000Total materials costs incurred for the month of $468,000 spread over 208,000 EUP results in a per-unit cost of $2.25.

Albany Mining Corporation uses a process costing system for its ore extraction operations. The following information pertains to work-in-process inventories and operations for the month of May: Completion % Units Materials Conversion WIP on May 1 32,000 60% 20% Started in production 200,000 Completed production 184,000 WIP on May 31 48,000 90% 40% Costs for the month were as follows: BWIP Incurred in May Materials $54,560 $ 468,000 Direct labor 20,320 182,880 Manufacturing overhead 15,240 391,160 $90,120 $1,042,040 Using the weighted-average method, Albany Mining's equivalent-unit cost of materials for May is A.$2.30 B.$2.25 C.$2.51 D.$2.06

A.$2.30Answer (A) is correct. The weighted-average method averages the work performed in the prior period with the work done in the current period.MaterialsUnits transferred out184,000Add: EWIP (48,000 × 90%)43,200EUP227,200The materials costs embedded in BWIP are combined with the materials costs incurred during the current period. The EUP materials cost is therefore $2.30 [($54,560 BWIP + $468,000 in May) ÷ 227,200 EU].

A company manufactures components for use in producing one of its finished products. When 12,000 units are produced, the full cost per unit is $35, separated as follows: Direct materials$ 5Direct labor15Variable overhead10Fixed overhead5 A supplier has offered to sell 12,000 components to the company for $37 each. If the company accepts the offer, some of the facilities currently being used to manufacture the components can be rented as warehouse space for $40,000. However, $3 of the fixed overhead currently applied to each component would have to be covered by the company's other products. What is the differential cost to the company of purchasing the components from the supplier? A.$20,000 B.$8,000 C.$24,000 D.$44,000

A.$20,000Answer (A) is correct. Differential (incremental) cost is the difference in total cost between two decisions. The relevant costs do not include unavoidable costs, such as the $3 of fixed overhead. It would cost the company an additional $20,000 to purchase, rather than manufacture, the components. Purchase price (12,000 × $37)$444,000Minus: rental income(40,000)Net cost to purchase $404,000Cost to manufacture (12,000 × $32)(384,000)Cost differential$ 20,000

A sporting goods manufacturer buys wood as a direct material for baseball bats. The Forming Department processes the baseball bats, and the bats are then transferred to the Finishing Department, where a sealant is applied. The Forming Department began manufacturing 10,000 "Casey Sluggers" during the month of May. There was no beginning inventory. Costs for the Forming Department for the month of May were as follows: Direct materials $33,000 Conversion costs 17,000 Total $50,000 A total of 8,000 bats were completed and transferred to the Finishing Department; the remaining 2,000 bats were still in the forming process at the end of the month. All of the Forming Department's direct materials were placed in process, but, on average, only 25% of the conversion cost was applied to the ending work-in-process inventory. The cost of the units transferred to the Finishing Department is A.$42,400 B.$50,000 C.$53,000 D.$40,000

A.$42,400Answer (A) is correct. The total EUP for materials equal 10,000 because all materials for the ending work-in-process had already been added to production. Thus, the materials cost per unit was $3.30 ($33,000 ÷ 10,000). For conversion costs, the total EUP equal 8,500 [8,000 completed + (2,000 in EWIP × 25%)]. Thus, the conversion cost was $2.00 per unit ($17,000 ÷ 8,500). The total cost transferred was therefore $42,400 [8,000 units × ($3.30 + $2.00)].

A company that produces a single product using a continuous process had no work in process on April 1. During the month of April, 10,000 units were started and 9,000 completed units were transferred. The ending work-in-process inventory was complete as to materials and 50% complete as to conversion. The cost of direct materials was $114,000, and the cost of direct labor amounted to $38,000. Manufacturing overhead is assigned at the rate of 50% of direct materials. For the purpose of determining the cost of goods manufactured in April, what is the cost per equivalent whole unit? A.$21.40 B.$15.40 C.$20.90 D.$23.22

A.$21.40Answer (A) is correct. The ending work-in-process inventory is 100% complete with respect to direct materials costs. Therefore, the direct materials cost per equivalent unit for the period is $11.40 ($114,000 ÷ 10,000). However, when it comes to conversion costs (direct labor and manufacturing overhead), the ending work-in-process inventory is only 50% complete. Including this incomplete ending inventory, which contains 1,000 units (10,000 started - 9,000 completed and transferred), the equivalent units of production for conversion costs total 9,500 units [9,000 completed and transferred out + (1,000 × 50%)]. As a result, the conversion costs per equivalent unit for the period total $10 {[$38,000 direct labor + ($114,000 × 50% application MOH rate)] ÷ 9,500 equivalent units}. Therefore, the total cost per equivalent unit is $21.40 ($11.40 + $10).

A company has two departments and allocates all overhead costs based on department revenues. The following information applies:Department 1 revenue$1,000,000Department 2 revenue1,500,000Corporate overhead costs250,000Department overhead costs175,000What amount is the company's total overhead cost for Department 2? A.$255,000 B.$105,000 C.$425,000 D.$170,000

A.$255,000Answer (A) is correct. Overhead costs are based on department revenues. Between Departments 1 and 2, there is total revenue of $2,500,000 ($1,000,000 + $1,500,000) and total costs of $425,000 ($250,000 + $175,000). Department 2 is responsible for 60% of total revenue ($1,500,000 ÷ $2,500,000) and therefore should be allocated 60% of the total overhead. Thus, Department 2's total overhead cost is $255,000 (60% × $425,000).

Spring Co. had two divisions: A and B. Division A created Product X, which could be sold on the outside market for $25 and used variable costs of $15. Division B could take Product X and apply additional variable costs of $40 to create Product Y, which could be sold for $100. Division B received a special order for a large amount of Product Y. If Division A was operating at full capacity, which of the following prices should Division A charge Division B for the Product X needed to fill the special order? A.$25 B.$15 C.$20 D.$40

A.$25Answer (A) is correct. Division A has no idle capacity. It will forgo $25 of revenue and a $10 contribution margin ($25 - $15 variable cost) for each unit of X transferred to Division B. Consequently, the price that optimizes decision making for the company as a whole is the $25 market price (assuming all the units transferred internally could be sold externally for $25). The true unit contribution margin for Y is $35 ($100 price - $40 variable cost of Y - $15 variable cost of X - $10 unit contribution margin of X forgone).

Trendy Co. produced and sold 30,000 backpacks during the last year at an average price of $25 per unit. Unit variable costs were the following: Variable manufacturing costs$ 9Variable selling and administrative costs6Total$15 Total fixed costs were $250,000. There was no year-end work-in-process inventory. If Trendy had spent an additional $15,000 on advertising, then sales would have increased by $30,000. If Trendy had made this investment, what change would have occurred in Trendy's pretax profit? A.$3,000 decrease. B.$4,200 increase. C.$4,200 decrease. D.$3,000 increase.

A.$3,000 decrease. Answer (A) is correct. Per-unit contribution margin is $10 ($25 selling cost - $15 variable cost). Selling 30,000 backpacks thus produces a total contribution margin of $300,000 (30,000 × $10), resulting in pretax profit of $50,000 (300,000 contribution margin - $250,000 fixed cost). If Trendy spends an additional $15,000 on advertising, sales increase by $30,000. This is an increase in unit sales of 1,200 backpacks ($30,000 sale increase ÷ $25 selling price). Therefore, if Trendy increases advertising expense, it will sell 31,200 (30,000 + 1,200) backpacks, producing a contribution margin of $312,000 (31,200 backpacks × $10 unit contribution margin) and a pretax profit of $47,000 ($312,000 contribution margin - $265,000 fixed cost). Profits would therefore decrease by $3,000 ($50,000 before - $47,000 after).

Oradell Company sells its single product at a price of $60 per unit and incurs the following variable costs per unit of product: Direct material $16 Direct labor 12 Manufacturing overhead 7 Variable manufacturing costs $35 Selling expenses 5 Total variable costs $40 Oradell's annual fixed costs are $880,000, and Oradell is subject to a 30% income tax rate. The annual sales revenue required by Oradell Company in order to achieve after-tax net income of $224,000 for the year is A.$3,600,000 B.$3,110,400 C.$1,656,000 D.$3,312,000

A.$3,600,000Answer (A) is correct. CVP analysis can be used to restate the equation for net income and determine the required level of unit sales.Target unit volume={Fixed costs + [Target net income ÷ (1.0 - tax rate)]} ÷ UCM={$880,000 + [$224,000 ÷ (1 - .30)]} ÷ ($60 - $40)=$1,200,000 ÷ $20=60,000 unitsAt a unit selling price of $60, the total revenue is $3,600,000 (60,000 units × $60).

Black Co.'s breakeven point was $780,000. Variable expenses averaged 60% of sales, and the margin of safety was $130,000. What was Black's contribution margin? A.$364,000 B.$546,000 C.$910,000 D.$1,300,000

A.$364,000Answer (A) is correct. The contribution margin equals total sales minus total variable costs. Thus, the contribution margin ratio (CMR) equals the contribution margin divided by total sales. The margin of safety is the excess of sales over breakeven sales. Black's total sales are therefore the sum of the breakeven amount of sales dollars and the margin of safety, or $910,000 ($780,000 + $130,000). The CMR equals 40% (1 - .6 average variable expenses). Accordingly, the contribution margin is $364,000 ($910,000 × 40%).

Clay Co. has considerable excess manufacturing capacity. A special job order's cost sheet includes the following applied manufacturing overhead costs:Fixed costs$21,000Variable costs33,000The fixed costs include a normal $3,700 allocation for in-house design costs, although no in-house design will be done. Instead, the job will require the use of external designers costing $7,750. What is the total amount to be included in the calculation to determine the minimum acceptable price for the job? A.$40,750 B.$36,700 C.$54,000 D.$58,050

A.$40,750Answer (A) is correct. Given excess capacity, neither increased fixed costs nor opportunity costs are incurred by accepting the special order. Thus, the marginal cost of the order (the minimum acceptable price) is $40,750 ($33,000 variable costs + $7,750 cost of external design).

The accountant for Champion Brake, Inc., applies overhead based on machine hours. The budgeted overhead and machine hours for the year are $260,000 and 16,000, respectively. The actual overhead and machine hours incurred were $275,000 and 20,000. The cost of goods sold and inventory data compiled for the year is as follows: Direct materials$ 50,000COGS450,000WIP (units)100,000Finished goods (units)150,000What is the amount of over/underapplied overhead for the year? A.$50,000 B.$15,000 C.$65,000 D.$67,000

A.$50,000Answer (A) is correct. The budgeted overhead application rate is $16.25 per machine hour ($260,000 budgeted total ÷ 16,000 budgeted hours). The amount of overhead applied was therefore $325,000 (20,000 actual machine hours × $16.25 budgeted rate). Actual overhead incurred was $275,000. Overhead was thus overapplied by $50,000 ($325,000 applied - $275,000 actual).

Richardson Motors uses 10 units of Part No. T305 each month in the production of large diesel engines. The cost to manufacture one unit of T305 is presented as follows: Direct materials $ 2,000 Materials handling (20% of direct materials cost) 400 Direct labor 16,000 Manufacturing overhead (150% of direct labor) 24,000 Total manufacturing cost $42,400 Materials handling, which is not included in manufacturing overhead, represents the direct variable costs of the receiving department that are applied to direct materials and purchased components on the basis of their cost. Richardson's annual manufacturing overhead budget is one-third variable and two-thirds fixed. Simpson Castings, one of Richardson's reliable vendors, has offered to supply T305 at a unit price of $30,000. Assume the rental opportunity does not exist and Richardson Motors could use the idle capacity to manufacture another product that would contribute $104,000 per month. If Richardson chooses to manufacture the ten T305 units in order to maintain quality control, Richardson's opportunity cost is A.$8,000 B.$(96,000) C.$88,000 D.$68,000

A.$8,000Answer (A) is correct. The out-of-pocket cost of making the part equals the total manufacturing cost minus the fixed overhead, or $26,400 {$42,400 - [(2 ÷ 3) × $24,000]}. The cost of the component consists of the $30,000 purchase price plus the $6,000 (20% of cost) of variable receiving costs, or a total of $36,000. Thus, unit out-of-pocket cost would increase by $9,600 if the components were purchased. For 10 units, the additional cost of purchasing is $96,000. However, the net effect of purchasing is a gain of $8,000 ($104,000 contribution from making another product - $96,000). Opportunity cost is the benefit from the next best alternative use of the resources. Hence, the company's opportunity cost of making the part is $8,000.

Brewster Co. has the following financial information:Fixed costs$20,000Variable costs60%Sales price$50What amount of sales is required for Brewster to achieve a 15% return on sales? A.$80,000 B.$133,333 C.$50,000 D.$33,333

A.$80,000Answer (A) is correct. If X equals sales, the sales required for a 15% return on sales is calculated as follows:X - .6X - $20,000=.15X.4X - $20,000=.15X.4X - .15X=$20,000.25X=$20,000X=$20,000 ÷ .25X=$80,000

Albany Mining Corporation uses a process costing system for its ore extraction operations. The following information pertains to work-in-process inventories and operations for the month of May: Completion % Units Materials Conversion WIP on May 1 32,000 60% 20% Started in production 200,000 Completed production 184,000 WIP on May 31 48,000 90% 40% Costs for the month were as follows: BWIP Incurred in May Materials $54,560 $ 468,000 Direct labor 20,320 182,880 Manufacturing overhead 15,240 391,160 $90,120 $1,042,040 Under the FIFO method, Albany Mining's equivalent units of production (EUP) with respect to materials are A.208,000 units. B.214,400 units. C.195,200 units. D.227,200 units.

A.208,000 units.Answer (A) is correct. Under the FIFO method, EUP are determined based only on work performed during the current period. Thus, units in beginning work-in-process must be backed out.MaterialsUnits transferred out184,000Add: EWIP (48,000 × 90%)43,200Total completed units227,200Minus: BWIP (32,000 × 60%)(19,200)Equivalent units of production208,000

Oradell Company sells its single product at a price of $60 per unit and incurs the following variable costs per unit of product: Direct material $16 Direct labor 12 Manufacturing overhead 7 Variable manufacturing costs $35 Selling expenses 5 Total variable costs $40 Oradell's annual fixed costs are $880,000, and Oradell is subject to a 30% income tax rate. If prime costs increased by 20% and all other values remained the same, Oradell Company's contribution margin (to the nearest whole percent) would be A.24% B.76% C.30% D.20%

A.24%Answer (A) is correct. Prime costs are direct materials and direct labor. Because their total was $28 ($16 + $12) before the increase, the new total is $33.60 ($28 × 1.2). Thus, prime costs increase by $5.60, and total variable costs increase to $45.60 ($40 + $5.60). Subtracting $45.60 from the $60 selling price leaves a contribution margin of $14.40. The contribution margin percentage thus becomes 24% ($14.40 ÷ $60).

A company manufactures a product that passes through two production departments: molding and assembly. Direct materials are added in the assembly department when conversion is 50% complete. Conversion costs are incurred uniformly. The activity in units for the assembly department during April is as follows: UnitsWork-in-process inventory, April 1(60% complete as to conversion costs)5,000Transferred in from molding department32,000Defective at final inspection (within normal limits)2,500Transferred out to finished goods inventory28,500Work-in-process inventory, April 30(40% complete as to conversion costs)6,000 The number of equivalent units for direct materials in the assembly department for April calculated on the weighted-average basis is A.31,000 units. B.26,000 units. C.28,500 units. D.34,000 units.

A.31,000 units.Answer (A) is correct. The weighted-average approach averages the costs in beginning work-in-process with those incurred during the period. Accordingly, the degree of completion of the BWIP is ignored in computing the EUP for direct materials. Direct materials EUP therefore consist of units transferred to finished goods (28,500) and units that failed inspection (2,500), or 31,000. Ending work-in-process inventory has not reached the point at which materials are added.

In an activity-based costing system, what should be used to assign a department's manufacturing overhead costs to products produced in varying lot sizes? A.Multiple cause-and-effect relationships. B.Relative net sales values of the products. C.A product's ability to bear cost allocations. D.A single cause-and-effect relationship.

A.Multiple cause-and-effect relationships.Answer (A) is correct. Instead of using a single allocation base for overhead, an ABC system determines the multiple activities associated with the incurrence of costs and then accumulates a cost pool for each activity using the appropriate activity base (cost driver). Consequently, overhead is assigned based on the multiple cause-and-effect relationships between activities and their cost drivers.

Kerner Manufacturing uses a process cost system to manufacture laptop computers. The following information summarizes operations relating to laptop computer model #KJK20 during the quarter ending March 31: Direct UnitsMaterialsWIP inventory, January 1100$70,000Started during the quarter500Completed during the quarter400WIP inventory, March 31200Costs added during the quarter$750,000 Beginning work-in-process inventory was 50% complete for direct materials. Ending work-in-process inventory was 75% complete for direct materials. Using the FIFO method, what were the equivalent units of production with regard to materials for the quarter ended March 31? A.500 B.550 C.600 D.450

A.500Answer (A) is correct. Under the FIFO method, equivalent units are determined based only on work performed during the current period. They include work performed to complete BWIP, work on units started and completed during the period, and work done on EWIP. Thus, total FIFO EUP of materials are BWIP100 units×50%=50Started and completed(400 - 100 in BWIP)300 units×100%=300EWIP200 units×75%=150Total EUP500

A company employs a process cost system using the first-in, first-out (FIFO) method. The product passes through both Department 1 and Department 2 in order to be completed. Units enter Department 2 upon completion in Department 1. Additional direct materials are added in Department 2 when the units have reached the 25% stage of completion with respect to conversion costs. Conversion costs are added proportionally in Department 2. The production activity in Department 2 for the current month was as follows:Beginning work-in-process inventory (40% complete with respect to conversion costs) 15,000Units transferred in from Department 1 80,000Units completed and transferred to finished goods 85,000Ending work-in-process inventory (20% complete with respect to conversion costs)10,000How many equivalent units for direct materials were added in Department 2 for the current month? A.70,000 units. B.90,000 units. C.80,000 units. D.85,000 units.

A.70,000 units.Answer (A) is correct. Beginning inventory is 40% complete. Thus, direct materials have already been added. Ending inventory has not reached the 25% stage of completion, so direct materials have not yet been added to these units. Thus, the EUP for direct materials calculated on a FIFO basis are equal to the units started and completed in the current period (85,000 units completed - 15,000 units in BWIP = 70,000 units started and completed).

Information related to the revenue and costs of a product are as follows:Total fixed cost per month$3,600Desired net income per month480Selling price per unit20Variable cost per unit15 What number of units above the breakeven level must be sold to earn the desired level of net income per month? A.96 B.816 C.24 D.720

A.96Answer (A) is correct. Target units to earn a desired level of income equal [(Fixed costs + Target income) ÷ Unit contribution margin]. Therefore, in order to reach $480 in net income for the month, 816 target units [($3,600 fixed costs + $480 income) ÷ ($20 price - $15 variable costs)] must be produced. The breakeven point equals (Fixed costs ÷ Unit contribution margin); therefore, the breakeven point equals 720 units [$3,600 fixed costs ÷ ($20 price - $15 variable costs)]. Therefore, 96 units (816 - 720) above the breakeven point must be sold to earn $480 per month.

Which costing method is permitted by the SEC for financial reporting? A.All of the answers are correct. B.Process costing to a split-off point and job-order costing afterwards. C.Process costing. D.Job-order costing.

A.All of the answers are correct.Answer (A) is correct. Both job-order costing and process costing are accepted by the SEC for financial reporting. Selection between them is based on the nature and process of the reporting company. Job-order costing is used when each end product is unique. Process cost accounting assigns costs to homogeneous products that are mass produced on a continuous basis (e.g., petroleum products, thread, and computer monitors).

In a make-versus-buy decision, the relevant costs include variable manufacturing costs as well as A.Avoidable fixed costs. B.Depreciation costs. C.Factory management costs. D.General office costs.

A.Avoidable fixed costs.Answer (A) is correct. The relevant costs in a make-versus-buy decision are those that differ between the two decision choices. These costs include any variable costs plus any avoidable fixed costs. Avoidable fixed costs will not be incurred if the "buy" decision is selected.

In the computation of manufacturing cost per equivalent unit, the weighted-average method of process costing considers A.Current costs plus cost of beginning work-in-process inventory. B.Current costs plus cost of ending work-in-process inventory. C.Current costs only. D.Current costs minus cost of beginning work-in-process inventory.

A.Current costs plus cost of beginning work-in-process inventory.Answer (A) is correct. The weighted-average method of process costing combines the costs of work done in the previous period and the current period. Thus, the cost of the EUP is equal to the current cost (current period) plus the cost of BWIP (previous period).

A manufacturing company has several product lines. Traditionally, it has allocated manufacturing overhead costs between product lines based on total machine hours for each product line. Under a new activity-based costing system, which of the following overhead costs would be most likely to have a new cost driver assigned to it? A.Employee benefits expense. B.Depreciation expense. C.Repair and maintenance expense. D.Electricity expense.

A.Employee benefits expense.Answer (A) is correct. Resource drivers are measures of the resources consumed by an activity. Because employee benefits expense has no relationship to the amount of machine hours used, it is probable that this overhead cost most likely will have a new cost driver assigned to it.

In a traditional job-order cost system, the issuance of indirect materials to a production department increases A.Factory overhead control. B.Factory overhead applied. C.Stores control. D.Work-in-process control.

A.Factory overhead control.Answer (A) is correct. As overhead is incurred, factory overhead control is debited and accounts payable, supplies, etc., are credited. When overhead is applied, work-in-process is debited and factory overhead applied is credited. The difference between the debited and credited amounts is over- or underapplied overhead.

In calculating the breakeven point for a multiproduct company, which of the following assumptions are commonly made when variable costing is used? I Sales volume equals production volume. II Variable costs are constant per unit. III A given sales mix is maintained for all volume changes. A.I, II, and III. B.I and III. C.I and II. D.II and III.

A.I, II, and III.Answer (A) is correct. Cost-volume-profit analysis assumes that costs and revenues are linear over the relevant range. It further assumes that total fixed costs and unit variable costs are constant. Thus, total variable costs are directly proportional to volume. CVP analysis also assumes that there is no material change in inventory (sales = production) and that the mix of products is constant (or that only one product is produced).

Leland Manufacturing uses 10 units of Part Number KJ37 each month in the production of radar equipment. The unit cost to manufacture 1 unit of KJ37 is presented below. Direct materials $ 1,000 Materials handling (20% of direct materials cost) 200 Direct labor 8,000 Manufacturing overhead (150% of direct labor) 12,000 Total manufacturing cost $21,200 Materials handling represents the direct variable costs of the Receiving Department that are applied to direct materials and purchased components on the basis of their cost. This is a separate charge in addition to manufacturing overhead. Leland's annual manufacturing overhead budget is one-third variable and two-thirds fixed. Scott Supply, one of Leland's reliable vendors, has offered to supply Part Number KJ37 at a unit price of $15,000. Assume Leland Manufacturing is able to rent all idle capacity for $25,000 per month. If Leland decides to purchase the 10 units from Scott Supply, Leland's monthly cost for KJ37 would A.Increase $23,000. B.Decrease $7,000. C.Change by some amount other than those given. D.Increase $48,000.

A.Increase $23,000.Answer (A) is correct. In addition to the $15,000 purchase price, the company would still incur $8,000 per unit of unavoidable (fixed) manufacturing overhead (2/3 of $12,000). The materials handling charge of 20% of the purchase price of components would add another $3,000 per unit ($15,000 × .2). Therefore, the unit cost of purchase would be $26,000 ($15,000 + $8,000 + $3,000). Purchasing would increase unit cost by $4,800 ($26,000 cost to purchase - $21,200 cost to manufacture), an increase of $48,000 per month (10 units × $4,800). However, the $25,000 of rental income would reduce the increase in net costs to $23,000 per month.

The breakeven point in units increases when unit costs A.Increase and sales price remains unchanged. B.Remain unchanged and sales price increases. C.Decrease and sales price remains unchanged. D.Decrease and sales price increases.

A.Increase and sales price remains unchanged.Answer (A) is correct. A BEP ratio can be increased either by raising the numerator or lowering the denominator. The breakeven point in units is calculated by dividing fixed costs by the unit contribution margin. If selling price is constant and costs increase, the unit contribution margin decreases. The effect is to decrease the denominator and increase the ratio.

Leland Manufacturing uses 10 units of Part Number KJ37 each month in the production of radar equipment. The unit cost to manufacture 1 unit of KJ37 is presented below. Direct materials $ 1,000 Materials handling (20% of direct materials cost) 200 Direct labor 8,000 Manufacturing overhead (150% of direct labor) 12,000 Total manufacturing cost $21,200 Materials handling represents the direct variable costs of the Receiving Department that are applied to direct materials and purchased components on the basis of their cost. This is a separate charge in addition to manufacturing overhead. Leland's annual manufacturing overhead budget is one-third variable and two-thirds fixed. Scott Supply, one of Leland's reliable vendors, has offered to supply Part Number KJ37 at a unit price of $15,000. If Leland purchases the KJ37 units from Scott, the capacity Leland used to manufacture these parts would be idle. Should Leland decide to purchase the parts from Scott, the unit cost of KJ37 would A.Increase by $4,800. B.Change by some amount other than those given. C.Decrease by $6,200. D.Decrease by $3,200.

A.Increase by $4,800.Answer (A) is correct. In addition to the $15,000 purchase price, the company would still incur $8,000 per unit of unavoidable (fixed) manufacturing overhead (2/3 of $12,000). The materials handling charge of 20% of the purchase price of components would add another $3,000 per unit ($15,000 × .2). Therefore, the unit cost of purchase would be $26,000 ($15,000 + $8,000 + $3,000), which is $4,800 more than the current cost to manufacture.

A company currently manufactures all component parts used in the manufacture of various hand tools. A handle is used in three different tools. The unit cost budget for 20,000 handles isDirect material$ .60Direct labor.40Variable overhead.10Fixed overhead.20Total unit cost$1.30 A parts manufacturer has offered to supply 20,000 handles to the company for $1.25 each, delivered. If the company currently has idle capacity that cannot be used, accepting the offer will A.Increase the handle unit cost by $.15. B.Decrease the handle unit cost by $.05. C.Increase the handle unit cost by $.05. D.Decrease the handle unit cost by $.15.

A.Increase the handle unit cost by $.15.Answer (A) is correct. Since the fixed cost will be incurred whether the company makes or buys the part, the relevant unit cost of making the part is the $1.10 variable cost ($1.30 - $.20 fixed overhead). The existence of idle capacity indicates that the firm has no opportunity cost to be considered in the calculation. Thus, accepting the offer would increase costs by $.15 ($1.25 - $1.10) per unit.

A company's approach to an insourcing vs. outsourcing decision A.Involves an analysis of avoidable costs. B.Should use absorption (full) costing. C.Depends on whether the company is operating at or below normal volume. D.Should use activity-based costing.

A.Involves an analysis of avoidable costs.Answer (A) is correct. Available resources should be used as efficiently as possible before outsourcing. If the total relevant costs of production are less than the cost to buy the item, it should be produced in-house. The relevant costs are those that can be avoided.

Which of the following is a disadvantage of using a process costing system versus job-order costing? A.It involves the calculation of stage of completion of goods-in-process and the use of equivalent units. B.It is difficult to ensure that materials and labor are accurately charged to each specific job. C.It is difficult to determine cost of goods sold when partial shipments are made before completion. D.It is expensive to use as a good deal of clerical work is required.

A.It involves the calculation of stage of completion of goods-in-process and the use of equivalent units.Answer (A) is correct. Process costing is used to assign costs to similar products that are mass produced on a continuous basis. Costs are accumulated for a cost object consisting of a large number of similar units of goods or services, work-in-process is stated in terms of equivalent units, and unit costs are established based on the average cost of all units. Process costing, therefore, involves the calculation of the stage of completion of goods-in-process, a necessary step in determining the number of equivalent units.

Which of the following qualitative factors favors the buy choice in an insourcing vs. outsourcing decision? A.Maintaining a long-run relationship with suppliers is desirable. B.Quality control is critical. C.Idle capacity is available. D.All of the answers are correct.

A.Maintaining a long-run relationship with suppliers is desirable.Answer (A) is correct. The maintenance of long-run relationships with suppliers may become paramount in a make-or-buy decision. Abandoning long-run supplier relationships may cause difficulty in obtaining needed parts when terminated suppliers find it advantageous not to supply parts in the future

In computing the current period's manufacturing cost per equivalent unit of production (EUP), the FIFO method of process costing considers current period costs A.Only. B.Plus cost of beginning work-in-process (BWIP) inventory. C.Less cost of beginning work-in-process (BWIP) inventory. D.Plus cost of ending work-in-process (EWIP) inventory.

A.Only.Answer (A) is correct. An EUP is a set of inputs required to manufacture one physical unit. Calculating EUP for each factor of production facilitates measurement of output and cost allocation when work-in-process exists. Under the FIFO assumption, only current-period costs are allocated between cost of goods manufactured and ending work in process because FIFO maintains beginning inventory costs completely separable from current-period costs. The cost per EUP considers only current-period costs.

Egan Co. owns land that could be developed in the future. Egan estimates it can sell the land for $1,200,000, net of all selling costs. If it is not sold, Egan will continue with its plans to develop the land. As Egan evaluates its options for development or sale of the property, what type of cost would the potential selling price represent in Egan's decision? A.Opportunity. B.Future. C.Sunk. D.Variable.

A.Opportunity.Answer (A) is correct. Opportunity cost is the maximum benefit forgone by using a resource for a given purpose and not for the best alternative. If Egan develops the land, it will not receive the revenue from the selling price.

Limitations of an activity-based costing system include which of the following? A.The expense of obtaining cost data is relatively high. B.Activity-based costing systems are less reliable. C.Control of overhead costs is enhanced. D.It eliminates arbitrary assignment of overhead costs.

A.The expense of obtaining cost data is relatively high.Answer (A) is correct. An ABC system should be used when a company has a high level of indirect costs. Under ABC, indirect costs are assigned to activities and are then allocated to final cost objects based on a cause-and-effect analysis. The limitation is that an ABC system is more complex and expensive than a traditional system that uses only one cost pool.B.Activity-based costing systems are less reliable.

The Chocolate Baker specializes in chocolate baked goods. The firm has long assessed the profitability of a product line by comparing revenues to the cost of goods sold. However, Barry Love, the firm's new accountant, wants to use an activity-based costing system that takes into consideration the cost of the delivery person. Listed below are activity and cost information relating to two of Chocolate Baker's major products. MuffinsCheesecakeRevenue$53,000$46,000Cost of goods sold26,00021,000Delivery activity:Number of deliveries15085Average length of delivery10 min.15 min.Cost per hour for delivery$20.00$20.00Using activity-based costing, which one of the following statements is correct? A.The muffins are $1,925 more profitable. B.The muffins have a higher profitability as a percentage of sales and therefore are more advantageous. C.The cheesecakes are $75 more profitable. D.The muffins are $2,000 more profitable.

A.The muffins are $1,925 more profitable.Answer (A) is correct. The first step is to calculate the gross margin on the two products: MuffinsCheesecakeRevenues$53,000$46,000Cost of goods sold(26,000)(21,000)Gross margin$27,000$25,000The next step is to calculate total delivery cost for each product:MuffinsCheesecakeNumber of deliveries15085Times: Minutes per delivery× 10× 15Total delivery minutes1,5001,275Divided by: Minutes per hour÷ 60÷ 60Total delivery hours25.0021.25Times: Delivery cost per hour× $20× $20Total delivery cost$500$425The operating profits on these two products, and the difference between them, can now be determined:Muffins($27,000 - $500)$26,500Cheesecake($25,000 - $425)(24,575)Excess$ 1,925

A table manufacturing company has the following cost structure for producing table tops.Unit CostsDirect materials$23.00Direct labor12.00Variable manufacturing overhead10.00Fixed manufacturing overhead17.00Variable administrative costs2.00Fixed administrative costs3.00Total unit costs$67.00Recently, the table manufacturer received an offer from a corporation to supply the table tops to the table manufacturer. The table manufacturer is considering buying the table tops instead of manufacturing them internally. Which one of the following statements is correct? A.The table manufacturer should accept the offer if it is less than $47.00 and the table manufacturer has excess manufacturing capacity. B.The table manufacturer should reject the offer if it is less than $47.00 and the table manufacturer has excess manufacturing capacity. C.The table manufacturer should accept the offer if it is $50.00 or more and the table manufacturer has excess manufacturing capacity. D.The table manufacturer should reject the offer if it is $50.00 or more.

A.The table manufacturer should accept the offer if it is less than $47.00 and the table manufacturer has excess manufacturing capacity.Answer (A) is correct. The total unit cost includes $20 of fixed costs ($17 + $3) that is not avoidable if the units are purchased. Moreover, the company has excess capacity. The opportunity cost of making the table tops is zero because no production will be displaced. Consequently, the relevant unit cost of making the table tops is the $47 unit variable cost, and the supplier's price must be less to justify the buy decision.

A company is offered a one-time special order for its product and has the capacity to take this order without losing current business. Variable costs per unit and fixed costs in total will be the same. The gross profit for the special order will be 10%, which is 15% less than the usual gross profit. What impact will this order have on total fixed costs and operating income? A.Total fixed costs do not change, and operating income increases. B.Total fixed costs do not change, and operating income does not change. C.Total fixed costs increase, and operating income increases. D.Total fixed costs increase, and operating income decreases.

A.Total fixed costs do not change, and operating income increases.Answer (A) is correct. Variable costs per unit and fixed costs in total do not change. Because the company has excess capacity, accepting a project with a positive gross profit margin increases operating income.

Breakeven analysis assumes that over the relevant range, A.Unit variable costs are unchanged. B.Unit revenues are nonlinear. C.Total fixed costs are nonlinear. D.Total costs are unchanged.

A.Unit variable costs are unchanged. Answer (A) is correct. Breakeven analysis assumes that unit selling price and unit variable costs are constant within the relevant range. It further assumes that costs and revenues are linear. Thus, total variable costs vary directly with output.

Each of the following should be considered in the selection of appropriate cost drivers for an activity-based costing system, except A.Volume-based production. B.Cost of measurement. C.Degree of correlation. D.Behavioral effects.

A.Volume-based production.Answer (A) is correct. In selecting cost drivers, management should consider the behavioral effects, the cost of measurement, and the degree of correlation between the cost driver and the actual assumption of overhead. Volume-based production is not a factor in selecting cost drivers for an activity-based costing system.

A company would most benefit from using an activity-based costing (ABC) system as opposed to a traditional costing system under which of the following conditions? A.When indirect costs are a high percentage of total costs. B.When batch-level and product-sustaining costs are immaterial. C.When different products use the different activities of the department in the same proportions. D.When each department within the company has a single activity.

A.When indirect costs are a high percentage of total costs.Answer (A) is correct. Activity-based costing is a response to the significant increase in the incurrence of indirect costs resulting from the rapid advance of technology.

In developing a predetermined factory overhead application rate for use in a process costing system, which of the following could be used in the numerator and denominator? NumeratorDenominator A. Numerator Actual factory overhead Denominator Estimated machine hours B. Numerator Estimated factory overhead Denominator Estimated machine hours C. Numerator Estimated factory overhead Denominator Actual machine hours D. Numerator Actual factory overhead Denominator Actual machine hours

B. Numerator Estimated factory overhead Denominator Estimated machine hours Answer (B) is correct. The predetermined overhead rate is calculated by dividing the estimated overhead (the numerator) by the estimated amount of the activity base (the denominator). The latter may be direct labor hours, direct labor dollars, machine hours, or some other reasonable base.

Jones Corp. had an opportunity to use its capacity to produce an extra 5,000 units with a contribution margin of $5 per unit, or to rent out the space for $10,000. What was the opportunity cost of using the capacity? A.$25,000 B.$10,000 C.$15,000 D.$35,000

B.$10,000Answer (B) is correct. Opportunity cost is the maximum benefit forgone by using a resource for a given purpose instead of for the best alternative. Receipt of rent of $10,000 is the benefit forgone by using the space for production.

The expected selling price for a new product is $19.00. Management's goal is to obtain a 20% contribution margin on all sales. If the new product has variable selling and distribution costs of $3.00 per unit, what is the product's target variable manufacturing cost? A.$18.20 B.$12.20 C.$12.80 D.$15.80

B.$12.20Answer (B) is correct. The contribution margin (CM) is equal to sales revenue minus variable costs. The contribution margin ratio (CMR) is equal to the CM divided by sales revenue. Given the per unit sales revenue of $19 and per unit variable selling and distribution costs of $3, the CM can be calculated as CM = $19 - (X + $3), where X represents the target variable manufacturing cost. Note that this can be further simplified to CM = $16 - X. Further, the CMR can be calculated as 20% = CM ÷ $19. Because the CM is a variable in both formulas, the formulas can be combined to produce 20% = ($16 - X) ÷ $19. Solving for X results in a target variable manufacturing cost of $12.20.

During Year 1, Tharp Lab supplied hospitals with a comprehensive diagnostic kit for $120. At a volume of 80,000 kits, Tharp had fixed costs of $1 million and a profit before income taxes of $200,000. Because of an adverse legal decision, Tharp's Year 2 liability insurance increased by $1.2 million over Year 1. Assuming the volume and other costs are unchanged, what should the Year 2 price be if Tharp is to make the same $200,000 profit before income taxes? A.$120.00 B.$135.00 C.$150.00 D.$240.00

B.$135.00Answer (B) is correct. CVP analysis can be used to restate the equation for operating income to determine the required level of unit sales:Tharp's Year 1 unit variable cost (UVC) can thus be calculated as follows: 80,000 units=($1,000,000 + $200,000)($120 - UVC)80,000 × ($120 - UVC)=$1,200,000$120 - UVC=$15UVC=$105The Year 2 unit selling price (USP) can now be derived:80,000 units=($1,000,000 + $200,000 + $1,200,000)(USP - $105)80,000 × (USP - $105)=$2,400,000USP - $105=$30USP=$135

A multiproduct company currently manufactures 30,000 units of Part 730 each month for use in production. The facilities now being used to produce Part 730 have fixed monthly overhead costs of $150,000 and a theoretical capacity to produce 60,000 units per month. If the company were to buy Part 730 from an outside supplier, the facilities would be idle and 40% of fixed costs would continue to be incurred. There are no alternative uses for the facilities. The variable production costs of Part 730 are $11 per unit. Fixed overhead is allocated based on planned production levels. If the company continues to use 30,000 units of Part 730 each month, it would realize a net benefit by purchasing Part 730 from an outside supplier only if the supplier's unit price is less than A.$12.00 B.$14.00 C.$13.00 D.$12.50

B.$14.00Answer (B) is correct. The relevant cost to the company has two components. One is its own variable production cost for Part 730 ($11). The other is that portion of fixed costs that are incurred if production is done in-house ($150,000 × 60% = $90,000); on a per-unit basis, this translates into $3 ($90,000 ÷ 30,000 = $3). The total threshold for the outside supplier's price is therefore $14 ($11 + $3).

Merry Co. has two major categories of manufacturing overhead: material handling and quality control. The costs expected for these categories for the coming year are as follows: Material handling$120,000Quality inspection200,000The plant currently applies overhead based on direct labor hours. The estimated direct labor hours are 80,000 per year. The plant manager is asked to submit a bid and assembles the following data on a proposed job:Direct materials$4,000Direct labor (2,000 hours)6,000What is the estimated product cost on the proposed job? A.$14,000 B.$18,000 C.$8,000 D.$10,000

B.$18,000Answer (B) is correct. Product costs (also called inventoriable costs) are capitalized as part of finished goods inventory, and eventually become a component of cost of goods sold. All manufacturing costs (direct materials, direct labor, and manufacturing overhead) must be treated as product costs. The plant applies overhead based on direct labor hours. The manufacturing overhead costs for the coming year equal $320,000 ($120,000 Material handling + $200,000 Quality inspection). Thus, overhead per direct labor hour equals $4.00 ($320,000 manufacturing overhead costs ÷ 80,000 direct labor hours). For the proposed job, direct materials and direct labor costs equal $10,000 ($4,000 + $6,000). Overhead applied to the job equals $8,000 (2,000 direct labor hours × $4.00 per hour). Thus, the estimated product cost on the proposed job equals $18,000 ($10,000 direct materials and labor + $8,000 overhead).

Match Co. manufactures a product with the following costs per unit, based on a maximum plant capacity of 400,000 units per year:Direct materials$ 60Direct labor10Variable overhead40Fixed overhead30Total$140 Match has a ready market for all 400,000 units at a selling price of $200 each. Selling costs in this market consist of $10 per unit shipping and a fixed licensing fee of $50,000 per year. Reno Co. wishes to buy 5,000 of these units on a special order. There would be no shipping costs on this special order. What is the lowest price per unit at which Match should be willing to sell the 5,000 units to Reno? A.$140 B.$190 C.$110 D.$200

B.$190Answer (B) is correct. In the absence of excess capacity, the minimum price accepted for a special order must recover (1) the opportunity cost of accepting the special order and (2) the variable costs associated with accepting the order. In this case, the shipping costs on the special order have been eliminated and therefore need not be considered in the price. However, the remaining costs must still be recovered. Therefore, the minimum price that should be charged on the special order is the normal price less the cost of shipping, or $190 ($200 - $10).

In a lean accounting environment, a company accepts a special order to make 200 units of a product each month for the next 2 months for $130 per unit. The company normally sells the unit for $170 per unit with variable costs per unit at $80. The company plans to use excess capacity. By what amount would this special order increase profit? A.$16,000 B.$20,000 C.$52,000 D.$36,000

B.$20,000Answer (B) is correct. When a manufacturer has excess production capacity, there is no opportunity cost involved when accepting a special order. The total increase in profit from the special order is calculated as follows:Sales price$130Variable costs$80Contribution margin per unit ($130 - $80)$50Profit for 400 units (400 × $50)$20,000

The breakeven point in units sold for Tierson Corporation is 44,000. If fixed costs for Tierson are equal to $880,000 annually and variable costs are $10 per unit, what is the contribution margin per unit for Tierson Corporation? A.$0.05 B.$20.00 C.$44.00 D.$88.00

B.$20.00Answer (B) is correct. The breakeven point in units is equal to the fixed costs divided by the unit contribution margin (UCM). Fixed costs ÷ UCM= Breakeven point in units$880,000 ÷ UCM= 44,000 unitsUCM= $20

During March, Bly Company's Department Y equivalent unit product costs, computed under the weighted-average method, were as follows: Materials$1Conversion3Transferred-in5 Materials are introduced at the end of the process in Department Y. There were 4,000 units (40% complete as to conversion costs) in WIP at March 31. The total costs assigned to the March 31 WIP inventory should be A.$28,800 B.$24,800 C.$27,200 D.$36,000

B.$24,800Answer (B) is correct. The unit costs of EUP under weighted-average are given. EWIP consists of 4,000 units 40% complete as to conversion costs (1,600 EUP). Given also that materials are added at the end of the process, no materials cost is included in EWIP. Thus, the EWIP reflects only transferred-in costs and conversion costs. Transferred-in (4,000 × $5)$20,000Conversion (1,600 × $3)4,800EWIP$24,800

Albany Mining Corporation uses a process costing system for its ore extraction operations. The following information pertains to work-in-process inventories and operations for the month of May: Completion % Units Materials Conversion WIP on May 1 32,000 60% 20% Started in production 200,000 Completed production 184,000 WIP on May 31 48,000 90% 40% Costs for the month were as follows: BWIP Incurred in May Materials $54,560 $ 468,000 Direct labor 20,320 182,880 Manufacturing overhead 15,240 391,160 $90,120 $1,042,040 Under the weighted-average method, Albany Mining's equivalent-unit conversion cost for May is A.$3.31 B.$3.00 C.$2.92 D.$3.10

B.$3.00Answer (B) is correct. The weighted-average method averages the work performed in the prior period with the work done in the current period.ConversionUnits transferred out184,000Add: EWIP (48,000 × 40%)19,200EUP203,200The conversion costs in BWIP are combined with the conversion costs incurred during the current period. The EUP conversion cost is therefore $3.00 [($20,320 DL in BWIP + $15,240 MOH in BWIP + $182,880 DL in May + $391,160 MOH in May) ÷ 203,200].

The following information pertains to Clove Co. for the month just ended: Budgeted sales$1,000,000Breakeven sales700,000Budgeted contribution margin600,000Cash flow breakeven200,000 Clove's margin of safety is A.$400,000 B.$300,000 C.$500,000 D.$800,000

B.$300,000Answer (B) is correct. The margin of safety measures the amount by which sales may decline before losses occur. It is the excess of budgeted or actual sales over the breakeven sales. Given that the budgeted sales are $1,000,000 and the breakeven sales are $700,000, the margin of safety is $300,000 ($1,000,000 - $700,000).

H Corp. produces premium office chairs. Due to a recent change in market share, H must decide whether to make or buy an order of 1,000 chairs. The materials cost of producing a chair is $20 per pound, the cost of direct labor is $40 per direct labor hour, and manufacturing overhead (100% variable) is allocated at a rate of $10 per chair. Fixed costs for the year are $5 per chair, of which 20% are avoidable. Each chair requires 2 pounds of materials and 1 hour of direct labor to complete. Assuming that H has no excess capacity, what are the relevant costs per chair? A.$71 B.$95 C.$90 D.$91

B.$95Answer (B) is correct. When no available capacity exists, all variable and fixed costs are relevant to the make-or-buy decision. The total relevant costs are therefore $95 per chair [(2 pounds of materials × $20) + (1 direct labor hour × $40) + $10 variable manufacturing overhead + $5 fixed costs].

The Cutting Department is the first stage of Mark Company's production cycle. BWIP for this department was 80% complete as to conversion costs. EWIP was 50% complete. Conversion costs in the Cutting Department for the month just ended were as follows: UnitsCCBeginning WIP25,000$ 22,000Units started and costs incurred during the month135,000143,000Units completed and transferred to next department during the month100,000 Using the FIFO method, what was the conversion cost of ending WIP in the Cutting Department? A.$22,000 B.$39,000 C.$33,000 D.$78,000

B.$39,000Answer (B) is correct. Under the FIFO method, EUP for a period include only the work done that period and exclude any work done in a prior period. EWIP is 60,000 units (25,000 units BWIP + 135,000 units started - 100,000 units completed). Units started and completed equal 75,000 (100,000 completed units - 25,000 units BWIP). The total of conversion cost EUP for the period is calculated below. Work Donein CurrentUnitsPeriodCC (EUP)BWIP25,00020%5,000Started & completed75,000100%75,000EWIP60,00050%30,000Total EUP110,000 The total of the conversion costs for the period is given as $143,000. Dividing by total EUP of 110,000 gives a unit cost of $1.30. Thus, the conversion cost of the EWIP inventory is $39,000 (30,000 EUP in EWIP × $1.30).

The following were among Gage Co.'s costs during the month just ended: Normal spoilage$ 5,000Freight out10,000Excess of actual manufacturing costs over standard costs20,000Standard manufacturing costs100,000Actual prime manufacturing costs80,000 Gage's actual manufacturing overhead was A.$45,000 B.$40,000 C.$55,000 D.$120,000

B.$40,000Answer (B) is correct. Factory (manufacturing) overhead consists of all costs other than direct materials and direct labor that are associated with the manufacturing process. Because the excess of actual manufacturing costs over standard costs is $20,000, and the standard costs are $100,000, the actual manufacturing costs are $120,000. However, because $80,000 of these costs are prime costs, the remainder is factory overhead. Thus, actual manufacturing overhead is therefore $40,000 ($120,000 - $80,000).

The following information pertains to a manufacturing company: Total sales$80,000Total variable costs20,000Total fixed costs30,000What is the breakeven level in sales dollars? A.$30,000 B.$40,000 C.$50,000D .$80,000

B.$40,000Answer (B) is correct. The breakeven point in sales dollars equals fixed costs divided by the contribution margin ratio (CMR). The CMR equals .75 [($80,000 sales - $20,000 variable costs) ÷ $80,000 sales]. Thus, the breakeven level in sales dollars is $40,000 ($30,000 ÷ .75).

Del Co. has fixed costs of $100,000 and breakeven sales of $800,000. What is its projected profit at $1,200,000 sales? A.$150,000 B.$50,000 C.$200,000 D.$400,000

B.$50,000Answer (B) is correct. Del's contribution margin ratio (CMR) can be calculated as follows:BEP in dollars=Fixed costs ÷ CMRBEP in dollars × CMR=Fixed costsCMR=Fixed costs ÷ BEP in dollars=$100,000 ÷ $800,000=12.5%Thus, at sales of $1,200,000, contribution margin is $150,000 ($1,200,000 × 12.5%), and profit is $50,000 ($150,000 contribution margin - $100,000 fixed costs).

Refrigerator Company manufactures ice-makers for installation in refrigerators. The costs per unit, for 20,000 units of ice-makers, are as follows. Direct materials $ 7 Direct labor 12 Variable overhead 5 Fixed overhead 10 Total costs $34 Cool Compartments, Inc., has offered to sell 20,000 ice-makers to Refrigerator Company for $28 per unit. If Refrigerator accepts Cool Compartments' offer, two alternatives are available for the ice-maker manufacturing plant: the plant can be idled or it can be retooled to produce water filtration units. If Refrigerator retools its existing plant, revenues from the sale of water filtration units are estimated at $80,000, with variable costs amounting to 60% of sales. In addition, $6 per unit of the fixed overhead associated with the manufacture of ice-makers could be eliminated. For Refrigerator Company to determine the most appropriate action to take in this situation, the total relevant costs of make vs. buy, respectively, are A.$680,000 vs. $440,000. B.$600,000 vs. $528,000. C.$648,000 vs. $528,000 .D.$600,000 vs. $560,000.

B.$600,000 vs. $528,000.Answer (B) is correct. Relevant costs are those that vary depending on the option chosen. Thus, $4 per unit of fixed overhead (20,000 units × $4 = $80,000) will be incurred regardless of the option chosen. This amount is irrelevant to the decision. The costs eliminated by buying ice-makers are the relevant costs of making the ice-makers. They equal $24 per unit ($7 + $12 + $5) of variable costs (20,000 units × $24 = $480,000) and $6 per unit of fixed costs (20,000 units × $6 = $120,000), a total of $600,000. If the ice-makers are bought, and the plant is retooled, the contribution margin from the sale of water filters is $32,000 ($80,000 × 40%). Thus, the $560,000 price of ice-makers (20,000 units × $28) is reduced by $32,000, giving relevant costs of buying of $528,000.

The New Wave Co. is considering a new method for allocating overhead to its two products, regular and premium coffee beans. Currently New Wave is using the traditional method to allocate overhead, in which the cost driver is direct labor costs. However, it is interested in using two different drivers: machine hours (MH) for separating and roasting beans, and pounds of coffee for packing and shipping. Machine hours for the current month are 700 hours, direct labor cost per pound of coffee is $1.25, and direct materials cost per pound of coffee is $1.50. There are 1,000 pounds of coffee packed and shipped for the current month. The following data are also available: RegularPremiumOverhead for the current month$5,000.00Cost pool for separating and roasting beans3,500.00150 MH 550 MH Cost pool for packing and shipping1,500.00500 lbs. 500 lbs. What is the total cost per pound for the premium coffee using the new activity-based costing method? A.$5.00 B.$9.75 C.$5.75 D.$7.75

B.$9.75Answer (B) is correct. Under ABC, indirect costs are assigned to cost pools and then allocated to end products. The total cost for separating and roasting the beans is $3,500. This amount is allocated to products based on machine hours. Production of premium coffee uses 550 of 700 total machine hours, so the separating and roasting cost allocated to it is $2,750 [$3,500 × (550 MH ÷ 700 MH)]. Packing and shipping costs are allocated based on pounds of coffee. Because the amounts in pounds of regular and premium coffee are the same, each is allocated $750 ($1,500 × 50%) of packing and shipping costs. Thus, the total overhead cost for the premium coffee is $3,500 ($2,750 + $750). The overhead cost per pound is $7 ($3,500 ÷ 500 pounds). Given a direct cost per pound of $2.75 ($1.25 direct labor + $1.50 direct materials), the total cost per pound is $9.75 ($7 + $2.75).

Wren Co. manufactures and sells two products with selling prices and variable costs as follows:ABSelling price$18.00$22.00Variable costs12.0014.00 Wren's total annual fixed costs are $38,400. Wren sells four units of A for every unit of B. If operating income last year was $28,800, what was the number of units Wren sold? A.6,000 B.10,500 C.9,600 D.5,486

B.10,500Answer (B) is correct. The contribution margins of the two products are $6 and $8, respectively ($18 - $12 and $22 - $14). The units sold can be calculated as follows: =($38,400 + $28,800) ÷ ($6A + $8B)$6A + $8B=$67,200$6(4B) + $8B=$67,200$32B=$67,200B=2,100 unitsBecause 4 units of A are sold for every unit of B, the volume of A was 8,400 units (2,100 × 4). Thus, the total number of units sold was 10,500 (8,400A + 2,100B).

Spark Co. buys cordless phones for $125 each and sells them for $200 each. Spark pays a sales commission of $25 per phone sold and monthly fixed costs are $3,000. Assuming Spark desired a profit of 10% of sales, how many units must Spark sell? A.600 B.100 C.200 D.400

B.100Answer (B) is correct. The contribution margin per unit is $50 ($200 - $125 - $25). The desired profit per unit is $20 ($200 × 10%). The following equation can be established: $50X - $3,000 = $20X, where X represents the number of units required to be sold. Solving for X results in 100 units.

Kew Co. had 3,000 units in work-in-process at April 1 that were 60% complete as to conversion cost. During April, 10,000 units were completed. At April 30, the 4,000 units in work-in-process were 40% complete as to conversion cost. Direct materials are added at the beginning of the process. How many units were started during April? A.9,800 B.11,000 C.10,000 D.9,000

B.11,000Answer (B) is correct. The following physical flow formula may be used to calculate the unknown:BWIP + Units started=Units completed + EWIP3,000 + Units started=10,000 + 4,000Units started=11,000

As part of the annual budgeting process, Fair Theatre Company compiled the following information for its next play production:Fixed expenses$48,000Variable expenses$10 per ticketTicket price$16How many tickets would Fair need to sell for the play's run to obtain a $24,000 profit? A.8,000 B.12,000 C.4,500 D.7,200

B.12,000Answer (B) is correct. An amount of operating income in dollars can be calculated by treating target income as an additional fixed cost.Target units=(Fixed costs + Target operating income) ÷ Unit contribution margin=($48,000 + $24,000) ÷ ($16 - $10)=12,000 units

During the current year, the following manufacturing activity took place for a company's products: Beginning work-in-process, 70% complete10,000 unitsUnits started into production during the year150,000 unitsUnits completed during the year140,000 unitsEnding work-in-process, 25% complete20,000 unitsWhat was the number of equivalent units produced using the first-in, first-out method (FIFO)? A.140,000 B.138,000 C.145,000 D.150,000

B.138,000Answer (B) is correct. The FIFO method considers only the work done in the current period in calculating EUP. The EUP needed to complete beginning work-in-process (BWIP) equal 3,000 [10,000 units × (100% - 70%)]. The EUP for units started and completed equal 130,000 [(140,000 units completed - 10,000 units in BWIP) × 100%]. The EUP for units in ending work-in-process (EWIP) equal 5,000 (20,000 units × 25%). Thus, total EUP equal 138,000.

The following data pertain to a company's cracking-department operations in December: UnitsCompletionWork-in-process, Dec. 120,00050%Units started170,000Units completed and transferred to the distilling department180,000Work-in-process, Dec. 3110,00050% Materials are added at the beginning of the process, and conversion costs are incurred uniformly throughout the process. Assuming use of the FIFO method of process costing, the equivalent units of production (EUP) with respect to conversion performed during December were A.180,000 B.175,000 C.185,000 D.170,000

B.175,000Answer (B) is correct. Under the FIFO method, EUP are determined based only on work performed during the current period. Thus, units in beginning work-in-process must be excluded.ConversionUnits transferred out180,000Add: EWIP (10,000 × 50%)5,000Total completed units185,000Less: BWIP (20,000 × 50%)(10,000)Equivalent units of production175,000

Albany Mining Corporation uses a process costing system for its ore extraction operations. The following information pertains to work-in-process inventories and operations for the month of May: Completion % Units Materials Conversion WIP on May 1 32,000 60% 20% Started in production 200,000 Completed production 184,000 WIP on May 31 48,000 90% 40% Costs for the month were as follows: BWIP Incurred in May Materials $54,560 $ 468,000 Direct labor 20,320 182,880 Manufacturing overhead 15,240 391,160 $90,120 $1,042,040 Under the FIFO method, Albany Mining's equivalent units of production (EUP) with respect to conversion costs are A.177,600 units. B.196,800 units. C.171,200 units. D.184,000 units.

B.196,800 units.Answer (B) is correct. Under the FIFO method, EUP are determined based only on work performed during the current period. Thus, units in beginning work-in-process must be excluded.ConversionUnits transferred out184,000Add: EWIP (48,000 × 40%)19,200Total completed units203,200Minus: BWIP (32,000 × 20%)(6,400)EUP196,800

At annual sales of $900,000, the Ebo product has the following unit sales price and costs: Sales price$20Prime cost6Manufacturing overhead:Variable1Fixed7Selling & admin. costs:Variable1Fixed3Total costs(18)Profit$ 2 What is Ebo's breakeven point in units? A.25,000 B.37,500 C.45,000 D.31,500

B.37,500Answer (B) is correct. The breakeven point in units is equal to the fixed costs divided by the unit contribution margin (UCM). The number of units sold was 45,000 ($900,000 revenues ÷ $20 unit sales price). Total fixed costs were thus $450,000 [(45,000 units × ($7 FOH + $3 FS&A)]. The UCM equals unit sales price minus unit variable cost ($20 - $6 - $1 - $1 = $12). Thus, the breakeven point in units is 37,500 ($450,000 fixed costs ÷ $12 UCM).

A direct labor overtime premium should be charged to a specific job when the overtime is caused by the A.Management's requirement that the job be completed before the annual factory vacation closure. B.Customer's requirement for early completion of the job. C.Increased overall level of activity. D.Management's failure to include the job in the production schedule.

B.Customer's requirement for early completion of the job.Answer (B) is correct. A direct labor overtime premium equals the excess of the overtime pay rate over the regular rate, multiplied by total overtime hours. It is ordinarily considered an indirect cost, charged to overhead, and thereby allocated to all jobs. The reason is that the association of an overhead premium with a specific job may be attributable solely to random scheduling. Accordingly, the incurrence of the premium is usually regarded as a function of an abnormally large production volume, that is, as a condition affecting all jobs. However, if the premium directly results from the demands of a specific job, it should be charged as a direct cost to that job.

A company produces and sells two products. The first product accounts for 75% of sales, and the second product accounts for the remaining 25% of sales. The first product has a selling price of $10 per unit, variable costs of $6 per unit, and allocated fixed costs of $100,000. The second product has a selling price of $25 per unit, variable costs of $13 per unit, and allocated fixed costs of $212,000. At the breakeven point, what number of units of the first product will have been sold? A.14,625 B.39,000 C.52,000 D.25,000

B.39,000Answer (B) is correct. To find the breakeven point when a company makes multiple products, total fixed costs are divided by the weighted-average unit contribution margin (UCM). Products 1 and 2 account for 75% and 25% of total unit sales, respectively. Thus, the weighted-average UCM for a composite unit (3 units for Product 1 + 1 unit for Product 2) is calculated as follows: ProductUCMUnits per Composite UnitWeighted-Average Composite UCM1($10 - $6) = $4×3=$122($25 - $13) = $12×1=12$24The total number of composite units is 13,000 [($100,000 FC of Product 1 + $212,000 FC of Product 2) ÷ $24 wt. avg. UCM]. The total units of products 1 and 2 are 39,000 and 13,000, respectively. Product 1: 13,000 × 3 = 39,000Product 2: 13,000 × 1 = 13,000

Oradell Company sells its single product at a price of $60 per unit and incurs the following variable costs per unit of product: Direct material $16 Direct labor 12 Manufacturing overhead 7 Variable manufacturing costs $35 Selling expenses 5 Total variable costs $40 Oradell's annual fixed costs are $880,000, and Oradell is subject to a 30% income tax rate. The number of units of product that Oradell Company must sell annually to break even is A.30,800 units. B.44,000 units. C.22,000 units. D.35,200 units.

B.44,000 units.Answer (B) is correct. The breakeven point in units equals fixed costs divided by the unit contribution margin (UCM). At a selling price of $60 per unit and with variable costs of $40 per unit, the unit contribution margin is $20. Thus, the breakeven point is 44,000 units ($880,000 ÷ $20).

Which of the following statements about activity-based costing (ABC) is false? A.Activity-based costing is useful for allocating manufacturing overhead costs. B.Activity-based costing is more likely to result in major differences from traditional costing systems if the firm manufactures only one product rather than multiple products. C.In activity-based costing, cost drivers are what cause costs to be incurred. D.Activity-based costing differs from traditional costing systems in that products are not cross-subsidized.

B.Activity-based costing is more likely to result in major differences from traditional costing systems if the firm manufactures only one product rather than multiple products.Answer (B) is correct. ABC determines the activities that will serve as cost objects and then accumulates a cost pool for each activity using the appropriate activity base (cost driver). It is a system that may be employed with job order or process costing methods. Thus, when only one product is produced, the allocation of costs to the product is trivial. All of the cost is assigned to the product. The particular method used to allocate the costs does not matter.

Which of the following cost methods is least likely to result in cost shifting in a multiproduct manufacturing environment? A.Traditional costing. B.Activity-based costing. C.Straight-line costing. D.Process costing.

B.Activity-based costing.Answer (B) is correct. Activity-based costing (ABC) is a response to the significant increase in the incurrence of indirect costs. Under ABC, indirect costs are assigned to activities and then rationally allocated to end products. In contrast with the traditional indirect cost allocation methods, allocation based on activities results in more accurate allocations of indirect costs. Thus, ABC is least likely to result in cost shifting, also known as peanut-butter costing that results in product-cost cross-subsidization.

In an insourcing vs. outsourcing decision, the decision process favors the use of total costs rather than unit costs. The reason is that A.Unit cost may be calculated based on different volumes. B.All of the answers are correct. C.Allocated costs may be included in the unit amounts. D.Irrelevant costs may be included in the unit amounts.

B.All of the answers are correct.Answer (B) is correct. Unit costs should be used with extreme care. In each situation, they may be calculated based on a different volume level from that anticipated, so comparability may be lost. Irrelevant costs included in the unit cost should be disregarded; only relevant costs should be included in the analysis. Allocated costs should also be ignored, and only the relevant costs that will change with the option chosen should be considered.

Costs relevant to an insourcing vs. outsourcing decision include variable manufacturing costs as well as A.Property taxes. B.Avoidable fixed costs. C.Factory depreciation. D.Factory management costs.

B.Avoidable fixed costs.Answer (B) is correct. Relevant costs are anticipated costs that will vary among the choices available. If two courses of action share some costs, those costs are not relevant because they will be incurred regardless of the decision made. Relevant costs include fixed costs that could be avoided if the items were purchased from an outsider.

The following data pertains to Blue Co.:ProductPer-Unit Data Sales PriceVariable CostA$70$42B 50 28C 60 32A customer of Blue is going to spend a total of $40,000 on one product only. Which of the following products would Blue prefer to sell to the customer? A.B. B.C. C.Either A or C. D.A.

B.C.Answer (B) is correct. One of the ways to determine which product to sell is by calculating the net profit from each product. The net profit from each product is calculated by multiplying the contribution margin (Sales - Variable costs) by the total number of products that it is possible to sell based on the limited spending of $40,000. The net profit from Product A is $16,000 [($40,000 ÷ $70) × ($70 - $42)]. The net profit from Product B is $17,600 [($40,000 ÷ $50) × ($50 - $28)]. The net profit from Product C is $18,667 [($40,000 ÷ $60) × ($60 - $32)]. Therefore, Blue will choose to sell Product C. Another way to determine which product to sell is by comparing how much profit each product yields per $1 spent on the product. Product C yields $0.467 ($28 contribution margin ÷ $60 sales price) per each dollar invested. This amount is greater than that for Product B ($0.44) and Product A ($0.40).

All of the following are likely to be used as a cost allocation base in activity-based costing except the A.Number of different materials used to manufacture the product. B.Cost of materials used to manufacture the product. C.Units of materials used to manufacture the product. D.Number of vendors supplying the materials used to manufacture the product.

B.Cost of materials used to manufacture the product.Answer (B) is correct. Activity-based costing is founded on the idea that drivers for indirect cost assignment should be based on some level of activity. Cost of materials does not directly reflect a level of a given activity.

The following information is available on Tackler Co.'s two product lines:ChairsTablesSales$180,000$48,000Variable costs(96,000)(30,000)Contribution margin84,00018,000 Fixed costs:Avoidable(36,000)(12,000)Unavoidable(18,000)(10,800)Operating income (loss)$ 30,000$ (4,800)Assuming Tackler discontinues the tables line and does not replace it, the company's operating income will A.Increase by $4,800. B.Decrease by $6,000. C.Decrease by $10,800. D.Increase by $6,000.

B.Decrease by $6,000.Answer (B) is correct. This disinvestment decision eliminates $12,000 of avoidable fixed cost and the $18,000 contribution to overhead. The net effect on pretax income is therefore a $6,000 decrease ($12,000 - $18,000).

When considering a special order that will enable a company to make use of currently idle capacity, which of the following costs is irrelevant? A.Direct labor. B.Depreciation. C.Materials. D.Variable overhead.

B.Depreciation.Answer (B) is correct. Because depreciation is expensed whether or not the special order is accepted, it is irrelevant to the decision. Only the variable costs are relevant.

To identify costs that relate to a specific product, an allocation base should be chosen that A.Does not have a cause-and-effect relationship. B.Has a cause-and-effect relationship. C.Considers direct materials and direct labor but not factory overhead. D.Considers variable costs but not fixed costs.

B.Has a cause-and-effect relationship.Answer (B) is correct. A cost allocation base is the means by which costs are allocated. The cost allocation base is some variable (activity) that has a strong correlation with the incurrence of cost by the cost objective. For example, direct labor hours is frequently used as a cost allocation base because indirect costs are often correlated with such activity.

A company that produces 10,000 units has fixed costs of $300,000, variable costs of $50 per unit, and a sales price of $85 per unit. After learning that its variable costs will increase by 20%, the company is considering an increase in production to 12,000 units. Which of the following statements is correct regarding the company's next steps? A.If production is increased to 12,000 units, profits will increase by $100,000. B.If production remains at 10,000 units, profits will decrease by $100,000. C.If production is increased to 12,000 units, profits will increase by $50,000. D.If production remains at 10,000 units, profits will decrease by $50,000.

B.If production remains at 10,000 units, profits will decrease by $100,000.Answer (B) is correct. Variable costs will increase by $10 per unit ($50 × 20%). If production remains at 10,000 units, the additional cost is $100,000 (10,000 units × $10). Thus, profits will decrease by $100,000 assuming (1) the sales price and fixed costs do not change and (2) all units are sold for that sales price. The new unit contribution margin (UCM) is $25 ($85 sales price - $50 original unit VC - $10 increase in unit VC). The contribution margin given production and sale of 10,000 units is $250,000 (10,000 units × $25 UCM). The result is a $50,000 loss ($300,000 FC - $250,000 contribution margin). The original UCM was $35 ($85 sales price - $50 unit VC). Given production and sale of 10,000 units, the contribution margin is $35,000 (10,000 × $35 UCM), a gross profit of $50,000 ($350,000 contribution margin - $300,000 FC).

One of the items served at a restaurant is pizza that is prepared and baked in the restaurant's kitchen. Each pizza requires $3 of ingredients, $2 of labor to prepare the ingredients, $1 of labor to bake the pizza, and $4 of allocated occupancy costs. The restaurant's supplier has offered to sell the restaurant pre-made pizzas, ready for baking, for $8 each. If the restaurant accepts the supplier's offer, the existing pizza preparation space will be converted into two additional dining tables that will increase profits by $42,000 per year by selling drinks, side dishes, and desserts. The restaurant expects to sell 15,000 pizzas annually with or without the additional dining tables. Should the restaurant accept the supplier's offer? A.Yes, because the restaurant's profits will increase by $12,000. B.No, because the restaurant's profits will decrease by $3,000. C.No, because the restaurant's profits will decrease by $27,000. D.Yes, because the restaurant's profits will increase by $57,000.

B.No, because the restaurant's profits will decrease by $3,000.Answer (B) is correct. If the restaurant does not accept the supplier's offer, it would cost $10 ($3 ingredient costs + $2 labor to prepare the ingredients + $1 labor to bake + $4 occupancy costs) to make each pizza. If the restaurant accepts the supplier's offer, it would cost the restaurant $13 ($8 cost of pre-made pizza + $1 labor to bake + $4 occupancy cost) to complete each pizza. If the restaurant sells 15,000 pizzas and accepts the offer, there will be an additional expense of $45,000 (15,000 × $3). Additionally, if the offer is accepted, profits will increase by $42,000 due to the newly converted dining tables. Therefore, the overall restaurant profit would decrease by $3,000 ($42,000 - $45,000), and the offer should be declined.

A company with three products classifies its costs as belonging to five functions: design, production, marketing, distribution, and customer services. For pricing purposes, all company costs are assigned to the three products. The direct costs of each of the five functions are traced directly to the three products. The indirect costs of each of the five business functions are collected into five separate cost pools and then assigned to the three products using appropriate allocation bases. The allocation base that will most likely be the best for allocating the indirect costs of the distribution function is A.Number of customer phone calls. B.Number of shipments. C.Number of sales persons. D.Dollar sales volume.

B.Number of shipments.Answer (B) is correct. The number of shipments is an appropriate cost driver. A cause-and-effect relationship may exist between the number of shipments and distribution costs.

Which costing method is an automobile manufacturer most likely to use? A.None of the answers are correct. B.Process costing to a split-off point and job-order costing afterwards. C.Process costing. D.Job-order costing.

B.Process costing to a split-off point and job-order costing afterwards.Answer (B) is correct. Job-order costing is used when each end product is unique. Process cost accounting assigns costs to relatively homogeneous products that are mass produced on a continuous basis (e.g., petroleum products, thread, and computer monitors). Generally, all automobiles are homogeneous before the split-off point. After this point, automobiles will need to be produced based on more specific customer requirements. Then, the final products are differentiated. An automobile manufacturer is most likely to use process costing to a split-off point and then job-order costing.

An equivalent unit of direct materials or conversion cost is equal to A.A unit of work-in-process inventory. B.The amount of direct materials or conversion cost necessary to complete one unit of production. C.The amount of direct materials or conversion cost necessary to start a unit of production in work-in-process. D.Fifty percent of the direct materials or conversion cost of a unit of finished goods inventory (assuming a linear production pattern).

B.The amount of direct materials or conversion cost necessary to complete one unit of production.Answer (B) is correct. EUP measure the amount of work performed in each production phase in terms of fully processed units during a given period. Incomplete units are restated as the equivalent amount of completed units. The calculation is made separately for direct materials and conversion cost (direct labor and overhead).

In an activity-based costing (ABC) system, cost reduction is accomplished by identifying and eliminating All Cost DriversNonvalue-AddingActivities A. All Cost Drivers Yes Nonvalue-Adding Activities Yes B. All Cost Drivers No Nonvalue-Adding Activities No C. All Cost Drivers No Nonvalue-Adding Activities Yes D. All Cost Drivers Yes Nonvalue-Adding Activities No

C. All Cost Drivers No Nonvalue-Adding Activities Yes Answer (C) is correct. An ABC system determines activities associated with the incurrence of costs and then accumulates a cost pool for each activity. It then identifies the driver(s) used to trace those costs to such cost objects as products or services. A driver is a factor that causes a change in a cost. Activities that do not add customer value are identified and eliminated to the extent possible. A clear understanding of what causes a cost (the driver) helps eliminate the nonvalue-adding activities. However, all drivers cannot be eliminated.

If the fixed costs attendant to a product increase while variable costs and sales price remain constant, what will happen to contribution margin (CM) and breakeven point (BEP)? CMBEP A. CMDecrease BEPIncrease B. CMIncrease BEPDecrease C. CMUnchanged BEPIncrease D. CMUnchanged BEPUnchanged

C. CMUnchanged BEPIncrease Answer (C) is correct. The BEP in units is equal to fixed costs divided by the UCM (unit selling price - variable cost per unit). Consequently, an increase in fixed costs has no effect on the CM but causes the BEP to increase; that is, more units must be sold to cover the increased fixed costs.

What is the normal effect on the numbers of cost pools and cost assignment bases when an activity-based cost (ABC) system replaces a traditional cost system? Cost PoolsCost Assignment Bases A. Cost PoolsNo effect Cost AssignmentBasesNo effect B. Cost PoolsIncrease Cost AssignmentBasesNo effect C. Cost PoolsIncrease Cost AssignmentBasesIncrease D. Cost PoolsNo effect Cost AssignmentBasesIncrease

C. Cost PoolsIncrease Cost AssignmentBasesIncrease Answer (C) is correct. In an ABC system, cost assignment is more precise than in traditional systems because activities rather than functions or departments are defined as cost objects. This structure permits assignment to more cost pools and the identification of a driver specifically related to each. A driver is a factor that causes a change in a cost. Thus, an ABC system uses more cost pools and cost assignment bases than a traditional system.

A process costing system was used for a department that began operations in January. Approximately the same number of physical units, at the same degree of completion, were in work-in-process at the end of both January and February. Monthly conversion costs are allocated between ending work-in-process and units completed. Compared with the FIFO method, would the weighted-average method use the same or a greater number of equivalent units to calculate the monthly allocations? Equivalent Units for WeightedAverage Compared with FIFO JanuaryFebruary A. Equivalent Units for Weighted Average Compared with FIFO January Same February Same B. Equivalent Units for Weighted Average Compared with FIFO January Greater number February Same C. Equivalent Units for Weighted Average Compared with FIFO January Same February Greater number D. Equivalent Units for Weighted Average Compared with FIFO January Greater number February Greater number

C. Equivalent Units for Weighted Average Compared with FIFO January Same February Greater number Answer (C) is correct. The weighted-average method calculates EUP by adding the EUP in ending work-in-process (EWIP) to the total of all units completed during the period, regardless of when they were started. The FIFO method determines EUP by subtracting the work done on the beginning work-in-process (BWIP) in the prior period from the weighted-average total. If the number of EUP in BWIP is zero, as it was for the month of January, the two methods produce the same result. Otherwise, the weighted-average computation is greater.

Information for the month of January concerning Department A, the first stage of Ogden Corporation's production cycle, is as follows: MaterialsConversionBWIP$ 8,000$ 6,000Current costs40,00032,000Total costs$ 48,000$38,000Equivalent units using weighted-average method100,00095,000Average unit costs$ 0.48$ 0.40Goods completed90,000 unitsEWIP10,000 units Materials are added at the beginning of the process. The ending work-in-process is 50% complete as to conversion costs. How would the total costs accounted for be distributed, using the weighted-average method? GoodsCompletedEnding Work-In-Process A. Goods Completed $79,200 Ending Work- In-Process $8,800 B. Goods Completed $88,000 Ending Work- In-Process $6,800 C. Goods Completed $79,200 Ending Work- In-Process $6,800 D. Goods Completed $86,000 Ending Work- In-Process $0

C. Goods Completed $79,200 Ending Work- In-Process $6,800 Answer (C) is correct. The weighted-average method combines the costs in BWIP with those for the current period. Materials are added at the beginning of the process, and conversion costs are assumed to be incurred uniformly. EUP and average-unit cost calculations were given.Completed goods:Materials (90,000 × $.48)$43,200Conversion costs (90,000 × $.40)36,000Cost of completed goods$79,200Given that EWIP is 50% complete as to conversion costs, 5,000 (10,000 × 50%) equivalent units of conversion cost are in ending inventory.EWIP:Materials (10,000 × $.48)$ 4,800Conversion costs (5,000 × $.40)2,000Cost of EWIP$ 6,800

A manufacturer produces hacksaw blades. Recently, the manufacturer has decided to enhance its product line and offer band blades. Two alternatives are being analyzed: purchase band blades overseas or produce them in-house. If the band blades are made in-house, the manufacturer will not be able to produce hacksaw blades, forgoing a $30,000 profit contribution. Revenue from sale of band blades$180,000 Outside purchase170,000 Direct material and labor100,000 Variable manufacturing overhead50,000 Avoidable fixed manufacturing overhead10,000 Calculate the incremental cost of making and purchasing band blades, respectively. MakingPurchasing A. Making$160,000 Purchasing$170,000 B. Making $20,000 Purchasing$10,000 C. Making$190,000 Purchasing $170,000 D. Making$20,000 Purchasing$40,000

C. Making$190,000 Purchasing $170,000 Answer (C) is correct. The incremental cost of making the band blades is $190,000 ($100,000 direct material and labor + $50,000 variable manufacturing overhead + $10,000 avoidable fixed manufacturing overhead + $30,000 forgone profit contribution). The incremental cost of purchasing the band blades is the $170,000 outside purchase cost.

Stewart Industries has been producing two bearings, components B12 and B18, for use in production. B12 B18 Machine hours required per unit 2.5 3.0 Standard cost per unit: Direct material $ 2.25 $ 3.75 Direct labor 4.00 4.50 Manufacturing overhead: Variable (See Note 1) 2.00 2.25 Fixed (See Note 2) 3.75 4.50 $12.00 $15.00 Stewart's annual requirement for these components is 8,000 units of B12 and 11,000 units of B18. Recently, Stewart's management decided to devote additional machine time to other product lines resulting in only 41,000 machine hours per year that can be dedicated to the production of the bearings. An outside company has offered to sell Stewart the annual supply of the bearings at prices of $11.25 for B12 and $13.50 for B18. Stewart wants to schedule the otherwise idle 41,000 machine hours to produce bearings so that the company can minimize its costs (maximize its net benefits). Note 1: Variable manufacturing overhead is applied on the basis of direct labor hours. Note 2: Fixed manufacturing overhead is applied on the basis of machine hours. The net benefit (loss) per machine hour that would result if Stewart accepts the supplier's offer of $13.50 per unit for Component B18 is A.$.50 B.Some amount other than those given. C.$(1.00) D.$(1.75)

C.$(1.00)Answer (C) is correct. The variable costs of producing B18 total $10.50 ($3.75 + $4.50 + $2.25). Thus, purchasing at $13.50 would result in a loss of $3 per bearing. Given that each bearing requires 3 hours of machine time, the loss is $1 per machine hour.

A tailor estimates that 60,000 special zippers will be used in the manufacture of men's jackets during the next year. A zipper supplier has quoted a price of $.60 per zipper. The tailor would prefer to purchase 5,000 units per month, but the supplier is unable to guarantee this delivery schedule. To ensure availability of these zippers, the tailor is considering the purchase of all 60,000 units at the beginning of the year. Assuming the tailor can invest cash at 8%, the tailor's opportunity cost of purchasing the 60,000 units at the beginning of the year is A.$2,640 B.$36,000 C.$1,320 D.$1,440

C.$1,320Answer (C) is correct. The cost of 60,000 zippers is $36,000 (60,000 × $.60). The monthly cost is $3,000 (5,000 × $.60). The company would like to purchase the items monthly, so it will invest at least $3,000 in January. Accordingly, the zippers to be used in January will be purchased at the first of the year even if no special purchase is made. Thus, the incremental advance purchase is only $33,000. Because the alternative arrangement involves a constant monthly expenditure of $3,000, the incremental investment declines by that amount each month (for example, January $36,000 - $3,000 = $33,000; February $33,000 - $3,000 = $30,000 . . . ; December $3,000 - $3,000 = $0). The result is that the average incremental investment for the year is $16,500 ($33,000 ÷ 2), and the opportunity cost of purchasing 60,000 units at the beginning of the year is $1,320 ($16,500 × 8%).

In order to increase the profit margin for a certain product, a company is planning to purchase a custom-made machine for $5,000,000. It is anticipated that the introduction of the new machine will reduce the product's variable costs of labor and maintenance by $5.50 per unit and $.95 per unit, respectively. The product manager estimates that 500,000 units of the product will be manufactured and sold each year with a product life cycle of two years, at which time the machine will be discarded with no salvage value. What is the company's total cost savings over the product's life cycle? A.$725,000 B.$6,450,000 C.$1,450,000 D.$3,225,000

C.$1,450,000Answer (C) is correct. A total of 1,000,000 units are expected to be sold over the life of the asset (500,000 per year × 2 years). The machine reduces the cost of each unit by $6.45 ($5.50 + $0.95). A total cost reduction of $6,450,000 results. After accounting for the initial investment, the total cost savings are $1,450,000 ($6,450,000 - $5,000,000).

Lucy Sportswear manufactures a specialty line of T-shirts using a job-order cost system. During March, the following costs were incurred in completing Job ICU2: direct materials, $13,700; direct labor, $4,800; administrative, $1,400; and selling, $5,600. Overhead was applied at the rate of $25 per machine hour, and Job ICU2 required 800 machine hours. If Job ICU2 resulted in 7,000 good shirts, the cost of goods sold per unit would be A.$6.50 B.$6.30 C.$5.50 D.$5.70

C.$5.50Answer (C) is correct. Cost of goods sold is based on the manufacturing costs incurred in production but does not include selling or general and administrative expenses. Manufacturing costs equal $38,500 [$13,700 DM + $4,800 DL + (800 hours × $25) OH]. Thus, per-unit cost is $5.50 ($38,500 ÷ 7,000 units).

Albany Mining Corporation uses a process costing system for its ore extraction operations. The following information pertains to work-in-process inventories and operations for the month of May: Completion % Units Materials Conversion WIP on May 1 32,000 60% 20% Started in production 200,000 Completed production 184,000 WIP on May 31 48,000 90% 40% Costs for the month were as follows: BWIP Incurred in May Materials $54,560 $ 468,000 Direct labor 20,320 182,880 Manufacturing overhead 15,240 391,160 $90,120 $1,042,040 Using the weighted-average method, Albany Mining's total cost of work-in-process inventory at May 31 is A.$154,800 B.$153,264 C.$156,960 D.$155,424

C.$156,960Answer (C) is correct. The weighted-average method averages the work performed in the prior period with the work done in the current period.MaterialsUnits transferred out184,000Add: EWIP (48,000 × 90%)43,200EUP227,200The materials costs embedded in BWIP are combined with the materials costs incurred during the current period. The EUP materials cost is therefore $2.30 [($54,560 BWIP + $468,000 in May) ÷ 227,200 EU]. ConversionUnits transferred out184,000Add: EWIP (48,000 × 40%)19,200EUP203,200The conversion costs in BWIP are combined with the conversion costs incurred during the current period. The EUP conversion cost is therefore $3.00 [($20,320 DL in BWIP + $15,240 MOH in BWIP + $182,880 DL in May + $391,160 MOH in May) ÷ 203,200]. The valuation of ending work-in-process inventory is calculated as follows:Materials:(48,000 physical units × 90%) × $2.30 =$ 99,360Conversion:(48,000 physical units × 40%) × $3.00 =57,600$156,960

The following information is available on Crain Co.'s two product lines:ChairsTablesSales$180,000$ 48,000Variable costs(96,000)(30,000)Contribution margin$ 84,000$ 18,000Fixed costs:Avoidable(36,000)(12,000)Unavoidable(18,000)(10,800)Operating income (loss)$ 30,000$ (4,800)Assuming the tables line is discontinued and the factory space previously used to make tables is rented for $24,000 per year, operating income will increase by what amount? A.$24,000 B.$13,200 C.$18,000 D.$28,800

C.$18,000Answer (C) is correct. When making a decision, the only relevant revenues and costs are those that will change depending on which choice is made. If the tables line is discontinued, the relevant amounts can be calculated as follows:Sales eliminated$(48,000)Variable costs eliminated30,000Avoidable fixed costs eliminated12,000Net loss from discontinuation$ (6,000)Rental income gained24,000Increase in operating income$ 18,000

A 20% target contribution margin is set for Duct, which is a new product with the following unit costs:Manufacturing costsVariable$12Fixed8Selling & admin. costsVariable$ 3Fixed5 What is Duct's target selling price? A.$25.00 B.$18.00 C.$18.75 D.$33.60

C.$18.75Answer (C) is correct. The contribution margin (CM) is equal to sales revenue minus variable costs. Furthermore, the contribution margin ratio (CMR) is equal to the CM divided by sales revenue. Duct's variable costs total $15 ($12 manufacturing + $3 selling and admin.) and the target CMR is 20%, thus the following equations can be derived:CM=X - $1520%=CM ÷ X where X is equal to sales revenue per unit. Because the CM is used in both formulas, the CMR formula can be rearranged to 20% = (X - $15) ÷ X. Solving for X results in a target selling price of $18.75.

Hamilton Company uses job-order costing. Manufacturing overhead is applied to production at a predetermined rate of 150% of direct labor cost. Any over- or underapplied overhead is closed to the cost of goods sold account at the end of each month. Additional information is available as follows: Job 101 was the only job in process at January 31, with accumulated costs as follows:Direct materials$4,000Direct labor2,000Applied manufacturing overhead3,000Total manufacturing costs$9,000 Jobs 102, 103, and 104 were started during February. Direct materials requisitions for February totaled $26,000. Direct labor cost of $20,000 was incurred for February. Actual manufacturing overhead was $32,000 for February. The only job still in process on February 28 was Job 104, with costs of $2,800 for direct materials and $1,800 for direct labor. Over- or underapplied manufacturing overhead should be closed to the cost of goods sold account at February 28 in the amount o of A.$1,700 underapplied. B.$700 overapplied. C.$2,000 underapplied. D.$1,000 overapplied.

C.$2,000 underapplied.Answer (C) is correct. The amount of over- or underapplied overhead is the difference between the actual overhead incurred and the overhead applied. The amount of overhead applied was $30,000 ($20,000 DL cost ×150%). The amount of overhead incurred was $32,000. Consequently, underapplied overhead of $2,000 ($32,000 actual - $30,000 applied) should be closed to COGS.

Company management would like to calculate the breakeven point in sales dollars of its lone product, a farm combine-planter. The following cost information is available:Combine-planter unit selling price$ 1,000Variable manufacturing expenses per unit800Other variable expenses100Fixed manufacturing expenses15,000Marketing expense (fixed)7,000What amount is the company's breakeven point in sales? A.$110,000 B.$70,000 C.$220,000 D.$150,000

C.$220,000Answer (C) is correct. The breakeven point in sales dollars equals fixed costs divided by the contribution margin ratio. The fixed costs equal $22,000 ($15,000 + $7,000), and the contribution margin ratio is .10 {[$1,000 - ($800 + $100)] ÷ $1,000}. Thus, the breakeven point is $220,000 ($22,000 ÷ .10).

A corporation has its own cafeteria with the following annual costs:Food$100,000Labor75,000Overhead110,000Total$285,000 The overhead is 40% fixed. Of the fixed overhead, $25,000 is the salary of the cafeteria supervisor. The remainder of the fixed overhead has been allocated from total company overhead. Assuming the cafeteria supervisor will remain and the corporation will continue to pay his/her salary, the maximum cost the corporation will be willing to pay an outside firm to service the cafeteria is A.$219,000 B.$285,000 C.$241,000 D.$175,000

C.$241,000Answer (C) is correct. Given that overhead is 40% fixed, $66,000 ($110,000 × 60%) is variable, and $44,000 is fixed. Of the latter amount, $25,000 is attributable to the supervisor's salary. The $19,000 remainder is allocated from total company overhead and is unavoidable. Assuming the company will continue to pay the supervisor's salary if an outside firm services the cafeteria, the total fixed overhead is not an avoidable (incremental) cost. Thus, the total avoidable cost of the cafeteria's operation is $241,000 ($100,000 food + $75,000 labor + $66,000 VOH). This amount is the savings from hiring an outside firm. Accordingly, it is also the maximum that the corporation should be willing to pay the outside firm.

A company's target gross margin is 40% of the selling price of a product that costs $178 per unit. The product's selling price should be A.$249.20 B.$445.00 C.$296.67 D.$284.80

C.$296.67Answer (C) is correct. Gross margin is calculated as [1 - (Unit cost ÷ Unit selling price)]. The company's target gross margin is 40%, and the unit cost is $178. After rearranging the gross margin equation, the unit selling price equals [Unit cost ÷ (1 - Gross margin)]. Thus, the product's selling price is $296.67 [$178 ÷ (1 - 40%)].

Nil Co. uses a predetermined factory overhead application rate based on direct labor cost. For the year ended December 31, Nil's budgeted factory overhead was $600,000, based on a budgeted volume of 50,000 direct labor hours, at a standard direct labor rate of $6 per hour. Actual factory overhead amounted to $620,000, with actual direct labor cost of $325,000. For the year, overapplied factory overhead was A.$50,000 B.$20,000 C.$30,000 D.$25,000

C.$30,000Answer (C) is correct. Nil Co. applies factory overhead using a predetermined overhead rate, based on direct labor cost. Overhead was budgeted for $600,000 based on a budgeted labor cost of $300,000 ($6 × 50,000 hrs.). Thus, $2 of overhead was applied for each $1 of labor. Given actual labor cost of $325,000, $650,000 ($2 × $325,000) of overhead was applied during the period. Actual overhead was $620,000, so $30,000 ($650,000 - $620,000) was overapplied.

Black, Inc., employs a weighted-average method in its process costing system. Black's work-in-process inventory on June 30 consists of 40,000 units. These units are 100% complete with respect to materials and 60% complete with respect to conversion costs. The equivalent unit costs are $5.00 for materials and $7.00 for conversion costs. What is the total cost of the June 30 work-in-process inventory? A.$480,000 B.$288,000 C.$368,000 D.$200,000

C.$368,000Answer (C) is correct. Because the weighted-average method of process costing is used, no distinction is made between current-period and prior-period costs. Given 40,000 physical units that are 100% complete with regard to materials, the EUP cost of materials is $200,000 [(40,000 × 100%) × $5]. Given also that these units are 60% complete with regard to conversion costs, the EUP cost of conversion is $168,000 [(40,000 × 60%) × $7]. The total cost of work-in-process given EUP data is therefore $368,000 ($200,000 + $168,000).

Leland Manufacturing uses 10 units of Part Number KJ37 each month in the production of radar equipment. The unit cost to manufacture 1 unit of KJ37 is presented below. Direct materials $ 1,000 Materials handling (20% of direct materials cost) 200 Direct labor 8,000 Manufacturing overhead (150% of direct labor) 12,000 Total manufacturing cost $21,200 Materials handling represents the direct variable costs of the Receiving Department that are applied to direct materials and purchased components on the basis of their cost. This is a separate charge in addition to manufacturing overhead. Leland's annual manufacturing overhead budget is one-third variable and two-thirds fixed. Scott Supply, one of Leland's reliable vendors, has offered to supply Part Number KJ37 at a unit price of $15,000. Assume that Leland Manufacturing does not wish to commit to a rental agreement but could use idle capacity to manufacture another product that would contribute $52,000 per month. If Leland elects to manufacture KJ37 in order to maintain quality control, Leland's opportunity cost is A.$(20,000) B.Some amount other than those given C.$4,000 D.$18,000

C.$4,000Answer (C) is correct. Opportunity cost is the maximum alternative earnings that might have been obtained if the productive good, service, or capacity had been applied to some alternative use. The additional total monthly cost of purchasing the component is $48,000. If the idle facilities could be used to produce a product contributing $52,000 per month, the net benefit opportunity cost of manufacture would be $4,000. The $48,000 additional total monthly cost is calculated by subtracting the variable cost of producing KJ37 from the variable cost of purchasing it. The total variable cost of purchasing a unit of KJ37 is $18,000 ($15,000 + $3,000), which includes $3,000 of materials handling cost ($15,000 × 20%). The total variable cost of producing KJ37 of $13,200 per unit ($1,000 + $200 + $8,000 + $4,000), which includes variable manufacturing overhead cost of $4,000 ($12,000 ÷ 3). This gives us an additional cost of $4,800 per unit ($18,000 - $13,200). Since Leland produces 10 units per month, the additional monthly cost of purchasing the units is $48,000 ($4,800 × 10).

Echo Company uses a normalized job costing system and applies factory overhead on the basis of machine hours. Echo's yearly profit plan disclosed anticipated factory overhead of $4,800,000 if 200,000 machine hours are worked. By year end, actual factory overhead charges and machine hours worked amounted to $4,730,000 and 215,000, respectively. What amount correctly states the factory overhead applied to Echo's actual year-end overhead? A.$70,000 overapplied. B.$360,000 overapplied. C.$430,000 overapplied. D.$430,000 underapplied.

C.$430,000 overapplied.Answer (C) is correct. The standard overhead rate is $24 per machine hour ($4,800,000 ÷ 200,000). Total overhead applied is the product of the standard overhead rate and the actual hours, which is $5,160,000 ($24 × 215,000). With an actual overhead of $4,730,000 and an applied overhead of $5,160,000, the company has overapplied its overhead by $430,000 ($5,160,000 - $4,730,000).

Birk Co. uses a job-order cost system. The following debits (credit) appeared in Birk's work-in-process account for the month just ended: DateDescriptionAmount1Balance$ 4,00031Direct materials24,00031Direct labor16,00031Factory overhead12,80031To finished goods(48,000)Birk applies overhead to production at a predetermined rate of 80% of direct labor cost. Job No. 5, the only job still in process at month end, has been charged with direct labor of $2,000. What was the amount of direct materials charged to Job No. 5? A.$8,800 B.$3,000 C.$5,200 D.$24,000

C.$5,200Answer (C) is correct. Total debits to WIP are $56,800 ($4,000 + $24,000 + $16,000 + $12,800). Given a credit of $48,000 for finished goods, EWIP is $8,800 ($56,800 - $48,000). Of this amount, $2,000 is direct labor and $1,600 ($2,000 × 80%) is overhead. Hence, the amount of direct materials charged to Job No. 5 was $5,200 ($8,800 - $2,000 - $1,600).

Jonathan Manufacturing adopted a job-costing system. For the current year, budgeted cost driver activity levels for direct labor hours and direct labor costs were 20,000 and $100,000, respectively. In addition, budgeted variable and fixed factory overhead costs were $50,000 and $25,000, respectively. Actual costs and hours for the year were as follows: Direct labor hours21,000Direct labor costs$110,000Machine hours35,000 For a particular job, 1,500 direct-labor hours were used. Using direct-labor hours as the cost driver, what amount of overhead should be applied to this job? A.$7,500 B.$3,214 C.$5,625 D.$5,357

C.$5,625Answer (C) is correct. To apply overhead to the job, both variable and fixed overhead must be properly allocated using standard rates and direct-labor hours as the cost driver. The pertinent information given for this year's budget includes direct-labor hours of 20,000 and the budgeted variable and fixed factory overhead of $50,000 and $25,000, respectively. The variable overhead cost per direct labor hour is $2.50 ($50,000 ÷ 20,000 DLH), and the fixed overhead per direct labor hour is $1.25 ($25,000 ÷ 20,000 DLH). Thus, the total standard overhead cost per direct labor hour is $3.75. The total overhead to be applied is $5,625 (1,500 DLH × $3.75).

On January 1, Maples had two jobs in process: #506 with assigned costs of $10,500 and #507 with assigned costs of $14,250. During January, three new jobs, #508 through #510, were started and three jobs, #506, #507, and #508, were completed. Materials and labor costs added during January were as follows: Job NumberMaterials Labor506$ 0$2,00050701,5005084,0003,6005093,8002,0005102,6003,100 Manufacturing overhead is assigned at the rate of 200% of labor. What is the January cost of goods manufactured and transferred from work-in-process? A.$35,850 B.$42,950 C.$50,050 D.$25,300

C.$50,050Answer (C) is correct. The cost of goods manufactured and transferred out of work-in-process during January is made up of four components:Materials added in January ($0 + $0 + $4,000)$ 4,000Labor added in January ($2,000 + $1,500 + $3,600)7,100Manufacturing overhead ($7,100 × 2)14,200Prior assigned costs ($10,500 + $14,250)24,750Total$50,050

Zeta Company is preparing its annual profit plan. As part of its analysis of the profitability of individual products, the controller estimates the amount of overhead that should be allocated to the individual product lines from the information given in the next column: Wall Specialty Mirrors Windows Units produced 25 25 Material moves per product line 5 15 Direct labor hours per unit 200 200 Budgeted materials handling costs $50,000 Under activity-based costing (ABC), Zeta's materials handling costs allocated to one unit of wall mirrors would be A.$1,500 B.$2,500 C.$500 D.$1,000

C.$500Answer (C) is correct. An activity-based costing (ABC) system allocates overhead costs on the basis of some causal relationship between the incurrence of cost and activities. Because the moves for wall mirrors constitute 25% (5 ÷ 20) of total moves, the mirrors should absorb 25% of the total materials handling costs. Thus, $12,500 ($50,000 × 25%) is allocated to mirrors. The remaining $37,500 is allocated to specialty windows. Dividing the $12,500 by 25 units produces a cost of $500 per unit of mirrors.

The data below pertain to two types of products manufactured by Cobb Corp. Fixed costs total $300,000 annually. The expected mix in units is 60% for product Y and 40% for product Z. Per Unit Sales Price Variable Costs Product Y $120 $ 70 Product Z 500 200 How much is Cobb's breakeven point in dollars? A.$400,000 B.$300,000 C.$544,000 D.$420,000

C.$544,000Answer (C) is correct. The BEP in units is equal to fixed costs divided by the unit contribution margin (UCM). The weighted-average UCM is $150, calculated as follows: Product YProduct ZSales price$120$500Minus: variable costs(70)(200)Contribution margin$ 50$300Times: mix ratio× 60%× 40%Weighted contribution margin$ 30$120 The BEP in units is 2,000 units ($300,000 fixed costs ÷ $150 UCM). The revenue (sales) mix will include 1,200 units of Y (2,000 × 60%) and 800 units of Z (2,000 × 40%). Hence, the BEP in dollars will be $544,000 [(1,200 × $120) + (800 × $500)].

Listed below are a company's monthly unit costs to manufacture and market a particular product.Manufacturing costs:Direct materials$2.00Direct labor2.40Variable indirect1.60Fixed indirect1.00Marketing costs:Variable2.50Fixed1.50 The company must decide to continue making the product or buy it from an outside supplier. The supplier has offered to make the product at the same level of quality that the company can make it. Fixed marketing costs would be unaffected, but variable marketing costs would be reduced by 30% if the company were to accept the proposal. What is the maximum amount per unit that the company can pay the supplier without decreasing operating income? A.$5.25 B.$7.75 C.$6.75 D.$8.50

C.$6.75Answer (C) is correct. The key to this question is, what costs will the company avoid if it buys from the outside supplier? It will no longer incur the $2.00 of direct materials, nor the $2.40 of direct labor, nor the $1.60 of variable overhead, nor $0.75 ($2.50 × 30%) of the variable marketing costs (regardless of whether the company makes or buys, it will still incur 70% of the variable marketing costs). The firm will therefore avoid costs of $6.75 ($2.00 + $2.40 + $1.60 + $0.75). Hence, it will at least break even by paying no more than $6.75.

Lucas Co. has a job-order cost system. For the month of April, the following debits (credits) appeared in the general ledger account, work-in-process:April1Balance$ 24,00030Direct materials80,00030Direct labor60,00030Factory overhead54,00030To finished goods(200,000)Lucas applies overhead to production at a predetermined rate of 90% based on direct labor cost. Job No. 100, the only job still in process at the end of April, has been charged with factory overhead of $4,500. The amount of direct materials charged to Job No. 100 was A.$5,000 B.$18,000 C.$8,500 D.$4,500

C.$8,500Answer (C) is correct. The ending balance in the WIP account is $18,000 ($24,000 + $80,000 + $60,000 + $54,000 - $200,000). This amount equals the $218,000 sum of all debits to the account minus the $200,000 credit. The $18,000 balance consists of materials, labor, and overhead for Job No. 100. Overhead is given as $4,500 (90% of direct labor cost). Direct labor is thus $5,000 ($4,500 ÷ .90), and the amount of direct materials is $8,500 ($18,000 - $5,000 DL - $4,500 OH).

A company sells DVD players for $200 per unit. The players have a unit variable cost of $160. The company estimates that it will sell one home entertainment system for every four DVD players sold. Home entertainment systems have a unit variable cost of $460 and sell for $600 per unit. The company's fixed costs are $90,000. Assuming that the sales mix estimate is correct, how many DVD players need to be sold for the company to break even? A.300 B.500 C.1,200 D.1,500

C.1,200Answer (C) is correct. When multiple products are involved, the breakeven point in units can be calculated as follows: Breakeven point = Total fixed costs ÷ Composite unit contribution margin (UCM) To calculate the composite UCM, add the CM of each product, weighted according to the product mix. In this case, the product mix is 4:1 in favor of DVD players. The DVD player has a contribution margin (CM) of $40 ($200 - $160) while the home entertainment system has a CM of $140 ($600 - $460). The UCM for DVD players is therefore $160 ($40 × 4) while the UCM for the home entertainment system is $140 ($140 × 1). The composite UCM is thus $300 ($160 + $140). The fixed costs total $90,000. The composite units to be sold is therefore 300 ($90,000 ÷ 300). Each composite unit contains 4 DVD players, therefore the number of DVD players to be sold is 1,200 (300 × 4).

Yarn Co.'s inventories in process were at the following states of completion at the end of the month: No. of UnitsPercent Complete10090%5080%20010% Equivalent units of production amounted to A.350 B.180 C.150 D.330

C.150Answer (C) is correct. The EUP are calculated as shown below. 100 units × 90% complete9050 units × 80% complete40200 units × 10% complete20EUP150

A company is reviewing its financial forecast. The selling price of the company's product is $22.50, the per unit contribution margin is $12.50, and total fixed costs are $175,000. How many units of the product must be sold to generate $50,000 of profit? A.17,500 B.22,500 C.18,000 D.14,000

C.18,000Answer (C) is correct. The company must sell enough units to cover fixed costs and generate the target profit (i.e., breakeven units plus units required to generate target profit). The number of units is calculated as follows: [(Fixed costs + Target profit) ÷ Unit contribution margin]. The company has fixed costs of $175,000, a target profit of $50,000, and a unit contribution margin of $12.50. Thus, the company must sell 18,000 units [($175,000 + $50,000) ÷ $12.50].

A ceramics manufacturer sold cups last year for $7.50 each. Variable costs of manufacturing were $2.25 per unit. The company needed to sell 20,000 cups to break even. Net income was $5,040. This year, the company expects the following changes: sales price per cup to be $9.00, variable manufacturing costs to increase 33.3%, fixed costs to increase 10%, and the income tax rate to remain at 40%. Sales in the coming year are expected to exceed last year's sales by 1,000 units. How many units does the company expect to sell this year? A.21,000 B.21,960 C.22,600 D.21,600

C.22,600Answer (C) is correct. The number of units the company expects to sell this year can be found once the number of units sold last year is derived. Fixed costs can be found using breakeven analysis.Breakeven point in units=Fixed costs (FC) ÷ Unit contribution margin (UCM)20,000 units=FC ÷ ($7.50 sales price - $2.25 unit variable manufacturing cost)20,000 units=FC ÷ $5.25 UCM$105,000=FCFixed costs can then be used to calculate the number of units sold last year.[($5.25 UCM × Units sold last year) - $105,000 FC] × (1 - .4)=$5,040 net income($5.25 UCM × Units sold last year) - $105,000 FC=$8,400$5.25 UCM × Units sold last year=$113,400Units sold last year=21,600The company expects to sell 1,000 more units this year than it did last year. Thus, it expects to sell 22,600 units.

A department adds material at the beginning of a process and identifies defective units when the process is 40% complete. At the beginning of the period, there was no work in process. At the end of the period, the number of work-in-process units equaled the number of units transferred to finished goods. If all units in ending work in process were 66% complete, then ending work in process should be allocated A.40% of all normal defective unit costs. B.None of the normal defective unit costs .C.50% of all normal defective unit costs. D.50% of the material costs and 40% of the conversion costs of all normal defective unit costs.

C.50% of all normal defective unit costs.Answer (C) is correct. Inspection occurs when the units are 40% complete. Thus, EWIP, which is 66% complete, contains good units only. Because normal spoilage attaches to good units, and the units transferred to finished goods equal those in EWIP, the normal defective unit costs should be allocated 50% to EWIP and 50% to finished goods.

Snyder Co. manufactures fans with direct material costs of $10 per unit and direct labor of $7 per unit. A local carrier charges Snyder $5 per unit to make deliveries. Sales commissions are paid at 10% of the selling price. Fans are sold for $100 each. Indirect factory costs and administrative costs are $6,800 and $37,200 per month, respectively. How many fans must Snyder produce to break even? A.1,375 B.564 C.648 D.530

C.648Answer (C) is correct. The breakeven point in units is derived by dividing fixed costs by the unit contribution margin. Snyder has fixed costs of $44,000 per month ($6,800 indirect factory costs + $37,200 administrative costs). The unit contribution margin is $100 - $10 - $7 - $5 - ($100 × 10%) = $68. The breakeven point is therefore $44,000 ÷ $68 = 647.06 units. However, Snyder cannot make a fractional unit, so it must produce 648 units.

When only differential manufacturing costs are taken into account for special-order pricing, an essential assumption is that A.Selling and administrative fixed and variable costs are linear. B.Manufacturing fixed and variable costs are linear. C.Acceptance of the order will not affect regular sales. D.Acceptance of the order will not cause unit selling and administrative variable costs to increase.

C.Acceptance of the order will not affect regular sales.Answer (C) is correct. Granting a lower-than-normal price for a special order has potential effects on regular sales. Other customers may demand the same price. Thus, the decision to consider only differential (marginal) manufacturing costs should be based on a determination that all other costs are not relevant. The other costs should not vary with the option chosen.

In an insourcing vs. outsourcing situation, which of the following qualitative factors is usually considered? A.Special materials requirements. B.Skilled labor. C.All of the answers are correct. D.Special technology.

C.All of the answers are correct.Answer (C) is correct. Special technology may be available either within or outside the firm that relates to the particular product. The firm may possess necessary skilled labor or the supplier may. Special materials requirements may also affect the decision process because one supplier may have monopolized a key component. Another factor to be considered is that assurance of quality control is often a reason for making rather than buying.

The following information is available on Tree Co.'s two product lines:LeavesBarkSales$ 30,000$ 50,000Variable cost(10,000)(25,000)Unavoidable fixed cost(10,000)(10,000)Operating income$ 10,000$ 15,000Assume that Tree Co. has the ability to discontinue both product lines and rent the factory space previously used for the production of Leaves and Bark. The factory space for Leaves can be rented out for $25,000 a year, and the factory space for Bark can be rented for $27,000 a year. Which product lines should Tree Co. discontinue and rent out instead? A.Only Leaves should be discontinued and rented out. B.Only Bark should be discontinued and rented out. C.Both Leaves and Bark should be discontinued and rented out. D.Neither Leaves nor Bark should be discontinued.

C.Both Leaves and Bark should be discontinued and rented out.Answer (C) is correct. Only the relevant costs of sales and variable costs should be considered when making the decision. The relevant amounts can be calculated as follows:LeavesBarkSales eliminated$(30,000)$(50,000)Variable costs eliminated10,00025,000Net loss from discontinuation(20,000)(25,000)Rental income gained25,00027,000Increase in operating income$ 5,000$ 2,000Leaves and Bark should be discontinued because the net loss for each is less than the annual rental income.

At the breakeven point, the contribution margin equals total A.Selling and administrative costs. B.Sales revenues. C.Fixed costs. D.Variable costs.

C.Fixed costs.Answer (C) is correct. No profit or loss occurs at the breakeven point. Thus, operating income equals zero, and fixed cost must equal the contribution margin (total revenue - total variable cost).

Cost allocation is the process of assigning indirect costs to a cost object. The indirect costs are grouped in cost pools and then allocated by a common allocation base to the cost object. The base that is employed to allocate a homogeneous cost pool should A.Be a nonfinancial measure (e.g., number of setups) because a nonfinancial measure is more objective. B.Assign the costs in the pool uniformly to cost objects even if the cost objects use resources in a nonuniform way. C.Have a cause-and-effect relationship with the cost items in the cost pool. D.Have a high correlation with the cost items in the cost pool as the sole criterion for selection.

C.Have a cause-and-effect relationship with the cost items in the cost pool.Answer (C) is correct. A cost allocation base is the common denominator for systematically correlating indirect costs and a cost object. The cost driver of the indirect costs is ordinarily the allocation base. In a homogeneous cost pool, all costs should have the same or a similar cause-and-effect relationship with the cost allocation base.

Richardson Motors uses 10 units of Part No. T305 each month in the production of large diesel engines. The cost to manufacture one unit of T305 is presented as follows: Direct materials $ 2,000 Materials handling (20% of direct materials cost) 400 Direct labor 16,000 Manufacturing overhead (150% of direct labor) 24,000 Total manufacturing cost $42,400 Materials handling, which is not included in manufacturing overhead, represents the direct variable costs of the receiving department that are applied to direct materials and purchased components on the basis of their cost. Richardson's annual manufacturing overhead budget is one-third variable and two-thirds fixed. Simpson Castings, one of Richardson's reliable vendors, has offered to supply T305 at a unit price of $30,000. Assume Richardson Motors is able to rent all idle capacity for $50,000 per month. If Richardson decides to purchase the 10 units from Simpson Castings, Richardson's monthly cost for T305 would A.Decrease $64,000. B.Decrease $14,000. C.Increase $46,000. D.Increase $96,000.

C.Increase $46,000.Answer (C) is correct. The out-of-pocket cost of making the part equals the total manufacturing cost minus the fixed overhead, or $26,400 {$42,400 - [(2 ÷ 3) × $24,000]}. The cost of the component consists of the $30,000 purchase price plus the $6,000 (20% of cost) of variable receiving costs, or a total of $36,000. Thus, unit out-of-pocket cost would increase by $9,600 if the components were purchased. For 10 components, the total cost increase would be $96,000, but the $50,000 rental would reduce the net increase to $46,000.

A company is considering outsourcing one of the component parts for its product. The company currently makes 10,000 parts per month. Current costs are as follows: Per unitTotalDirect materials$4$40,000Direct labor330,000Fixed plant facility cost220,000The company decides to purchase the part for $8 per unit from another supplier and rents its idle capacity for $5,000/month. How will the company's monthly costs change? A.Decrease $15,000. B.Increase $10,000. C.Increase $5,000. D.Decrease $10,000.

C.Increase $5,000.Answer (C) is correct. If the company were to purchase the part instead of making it, it would save both direct materials ($40,000) and direct labor ($30,000) costs. However, it would still incur the fixed plant facility cost ($20,000) in addition to the purchase price of the parts themselves (10,000 units × $8 purchase price = $80,000). This total can be reduced by the $5,000 income the company would earn by renting its idle capacity. Therefore, the total cost to purchase the component is $95,000 ($20,000 + $80,000 - $5,000). The total cost of manufacturing in-house is $90,000 ($40,000 direct materials + $30,000 direct labor + $20,000 fixed plant facility cost). If the company purchases the part from outside, its monthly costs will therefore increase by $5,000 ($95,000 to purchase - $90,000 to produce).

Purchased direct materials are added in the second department of a three-department process. This addition does not increase the number of units produced in the second department and will A.Not change the dollar amount transferred to the next department. B.Increase the factory overhead portion of the ending work-in-process inventory. C.Increase total unit cost. D.Decrease total ending work-in-process inventory.

C.Increase total unit cost.Answer (C) is correct. Adding materials to a production process without changing the number of units produced increases the unit cost. The numerator (total cost) increases while the denominator (total units) remains the same.

Which of the following is true about activity-based costing? A.It can be used only with process costing. B.It can be used only with job costing. C.It can be used with either process or job costing. D.It should not be used with process or job costing.

C.It can be used with either process or job costing.Answer (C) is correct. Activity-based costing may be used by manufacturing, service, or retailing entities and in job-order or process costing systems.

Assuming no beginning work-in-process (BWIP) inventory, and that the ending work-in-process (EWIP) inventory is 50% complete as to conversion costs, the number of equivalent units as to conversion costs would be A.The same as the units placed in process. B.Less than the units completed. C.Less than the units placed in process. D.The same as the units completed.

C.Less than the units placed in process.Answer (C) is correct. Given no BWIP, it is immaterial whether the FIFO or weighted-average method is used. Thus, conversion cost EUP equal the units that were started and completed this period plus the EUP in EWIP. Because the units in EWIP are 50% complete as to conversion costs, they will not be fully counted for purposes of determining EUP.

In using cost-volume-profit analysis to calculate expected unit sales, which of the following should be added to fixed costs in the numerator? A.Predicted operating loss. B.Variable costs. C.Predicted operating income. D.Unit contribution margin.

C.Predicted operating income.Answer (C) is correct. CVP analysis can be used to restate the equation for target net income to determine the required level of unit sales.

Which costing method is a gravel pit operation most likely to use? A.Process costing to a split-off point and job-order costing afterwards. B.None of the answers are correct. C.Process costing. D.Job-order costing.

C.Process costing.Answer (C) is correct. Job-order costing is used when each end product is unique. Process cost accounting assigns costs to inventoriable goods or services. It applies to relatively homogeneous products that are mass produced on a continuous basis (e.g., petroleum products, thread, and computer monitors). A gravel pit operation produces homogeneous end products on a continuous basis that are inventoriable. Thus, it is most likely to use process costing.

Which costing method is most likely to distort costs? A.Process costing to a split-off point and job-order costing afterwards. B.None of the answers are correct. C.Process costing. D.Job-order costing.

C.Process costing.Answer (C) is correct. Process cost accounting assigns costs to inventoriable goods or services. It applies to relatively homogeneous products that are mass produced on a continuous basis (e.g., petroleum products, thread, and computer monitors). Instead of using subsidiary ledgers to track specific jobs, process costing typically uses only a work-in-process account for each department through which the production of output passes. Thus, all costs incurred in each department are equally allocated on each product. If the products are not exactly the same, the costs allocated are most likely to be distorted under process costing.

A company plans to sell 12,000 units of product XT and 8,000 units of product RP. The company has a capacity of 12,000 productive machine hours. The unit cost structure and machine hours required for each product are as follows:Unit costs:XTRPMaterials$37$24Direct labor1213Variable overhead63Fixed overhead3738Machine hours required1.01.5The company can purchase 12,000 units of XT at $60 and/or 8,000 units of RP at $45. Based on the above, which one of the following actions should be recommended to the company's management? A.Purchase both XT and RP. B.Produce RP internally and purchase XT. C.Produce XT internally and purchase RP. D.Produce both XT and RP.

C.Produce XT internally and purchase RP.Answer (C) is correct. Relevant unit cost to product XT is $55 ($37 + $12 + $6) and cost for RP is $40 ($24 + $13 + $3). Only enough machine hours are available to produce one product or the other (XT: 12,000 × 1.0 hour per unit = 12,000 hours, RP: 8,000 units × 1.5 hours per unit = 12,000 hours). Cost to produce XT in-house would be $660,000 (12,000 × $55) and cost to produce RP would be $320,000 (8,000 × $40). Cost to purchase XT from outside would be $720,000 (12,000 × $60) and cost to purchase RP would be $360,000 (8,000 × $45). The possible combination with the lowest cost is to produce XT and purchase RP ($660,000 + $360,000 = $1,020,000).

A basic assumption of activity-based costing (ABC) is that A.All manufacturing costs vary directly with units of production. B.Only variable costs are included in activity cost pools. C.Products or services require the performance of activities, and activities consume resources. D.Only costs that respond to unit-level drivers are product costs.

C.Products or services require the performance of activities, and activities consume resources.Answer (C) is correct. ABC identifies activities needed to provide products or services, assigns costs to those activities, and then reassigns costs to the products or services based on their consumption of activities. ABC helps to manage costs by providing more detailed analyses of costs than traditional methods. It also facilitates cost reduction by determining what activities do and do not add value to the product or service.

A CPA would recommend implementing an activity-based costing system under which of the following circumstances? A.The client is a single-product manufacturer. B.The client produced many different products that homogeneously consume resources. C.The client produced products that heterogeneously consume resources. D.Most of the client's costs currently are classified as direct costs.

C.The client produced products that heterogeneously consume resources.Answer (C) is correct. Traditional cost systems allocate indirect costs by using a single volume-based cost driver. This may create significant inaccuracies when products do not use similar amounts of resources, and the overhead costs do not fluctuate with volume. The solution is an ABC system that assigns indirect costs to activities that are then rationally allocated to end products.

Which of the following changes would cause a company's breakeven point in sales to increase? A.The company's contribution-margin rate increases. B.The company's variable cost per unit decreases. C.The company's total fixed costs increases. D.The company's selling price per unit increases.

C.The company's total fixed costs increases.Answer (C) is correct. The breakeven point is the level of output at which all fixed costs and cumulative variable costs have been covered. It is the output at which operating income is zero. The breakeven point in sales dollars equals fixed costs divided by the contribution margin ratio (CMR). CMR is the ratio of contribution margin to sales price on either a total or per-unit basis. When total fixed costs increase, the breakeven point in sales increases, since a higher amount of revenue is necessary to cover those costs to breakeven.

Assuming no beginning work-in-process inventory, and that the ending work-in-process inventory is 100% complete as to materials costs, the number of equivalent units as to materials costs is A.The same as the units completed. B.Less than the units placed in process. C.The same as the units placed in process. D.Less than the units completed.

C.The same as the units placed in process.Answer (C) is correct. Given no BWIP, whether the FIFO or weighted-average method is used is immaterial. Because EWIP is 100% complete as to materials costs, the EUP for materials costs are equal to the number of units placed in process (units in EWIP + units transferred to finished goods).

Mason Co. uses a job-order cost system and applies manufacturing overhead to jobs using a predetermined overhead rate based on direct-labor dollars. The rate for the current year is 200% of direct-labor dollars. This rate was calculated last year and will be used throughout the current year. Mason had one job, No. 150, in process at the beginning of the month with raw materials costs of $2,000 and direct-labor costs of $3,000. During the month, raw materials and direct labor added to jobs were as follows: No. 150No. 151No. 152Raw materials$ -- $4,000$1,000Direct labor1,500 5,000 2,500 Actual manufacturing overhead for the month was $20,000. During the month, Mason completed Job Nos. 150 and 151. For the month, manufacturing overhead was A.Underapplied by $7,000 .B.Overapplied by $4,000. C.Underapplied by $2,000 .D.Underapplied by $1,000.

C.Underapplied by $2,000.Answer (C) is correct. Mason incurred direct-labor costs of $9,000 ($1,500 Job 150 + $5,000 Job 151 + $2,500 Job 152). Hence, overhead applied was $18,000 ($9,000 × 200%). The amount underapplied was $2,000 ($20,000 actual OH - $18,000).

A start-up firm has the following data: Date Units Purchased Price per Unit Total 1/5 100 $2 $ 200 2/5 150 3 450 3/7 125 4 500 Total 375 $1,150 Ending inventory is 140 units. If the firm wants to spread inventory costs across accounting periods (called peanut-butter costing) to stabilize working capital, which method should it choose? A.FIFO. B.Replacement cost. C.Weighted average. D.LIFO.

C.Weighted average.Answer (C) is correct. The weighted-average costing method spreads total costs over all units and results in less variable amounts of working capital. MethodCOGSEnding InventoryFIFO$605$545LIFO830320Weighted average721429Replacement cost940560

Lake Co. has just increased its direct labor wage rates. All other budgeted costs and revenues were unchanged. How did this increase affect Lake's budgeted breakeven point and budgeted margin of safety? BudgetedBreakeven PointBudgetedMargin of Safety A. Budgeted Breakeven Point Decrease Budgeted Margin of Safety Increase B. Budgeted Breakeven Point Decrease Budgeted Margin of Safety Decrease C. Budgeted Breakeven Point Increase Budgeted Margin of Safety Increase D. Budgeted Breakeven Point Increase Budgeted Margin of Safety Decrease

D. Budgeted Breakeven Point Increase Budgeted Margin of Safety Decrease Answer (D) is correct. The BEP is the sales volume at which total revenue equals total cost. The margin of safety is the excess of budgeted sales over the breakeven volume. Given that all other costs and revenues are constant, an increase in direct labor cost will increase the BEP and decrease the margin of safety.

The units transferred in from the first department to the second department should be included in the computation of the equivalent units for the second department under which of the following methods of process costing? FIFOWeighted-Average A. FIFOYes Weighted-AverageNo B. FIFONo Weighted-AverageNo C. FIFONo Weighted-AverageYes D. FIFOYes Weighted-AverageYes

D. FIFOYes Weighted-AverageYes Answer (D) is correct. The units transferred from the first to the second department should be included in the computation of EUP for the second department regardless of the cost flow assumption used. The transferred-in units are considered materials added at the beginning of the period.

The completion of goods is recorded as a decrease in work-in-process control when using Job-Order CostingProcess Costing A. Job-Order Costing No Process Costing No B. Job-Order Costing Yes Process Costing No C. Job-Order Costing No Process Costing Yes D. Job-Order Costing Yes Process Costing Yes

D. Job-Order Costing Yes Process Costing Yes Answer (D) is correct. The cost flow among accounts in process costing is similar to that for job-order costing. Both use the basic general ledger accounts, for example, materials control, work-in-process control, manufacturing overhead control, finished goods control, and cost of goods sold. Consequently, each system credits (decreases) work-in-process control and debits (increases) finished goods control when goods are completed.

A company needs special gears. The machinery to make the gears can be rented for $100,000 for 1 year, but the company can buy the gears and avoid the rental cost. Because the demand for the gears may be high (0.6 probability) or low (0.4 probability) and contribution margins vary, the company prepared the following decision tree: Which of the following statements is true? A.Making the gears is the best choice. B. The expected value of making is $20,000. C.Buying the gears is the best choice. D. The expected value of buying is $70,000.

D. The expected value of buying is $70,000.Answer (D) is correct. The expected value of buying the gears is.6×$100,000=$60,000.4×$ 25,000=10,000$70,000

A company uses process costing to assign product costs. Available inventory information for a period is as follows: Inventory (in units)Material costConversion costBeginning0Started during the period15,000$75,000$55,500Transferred out13,500End of period1,500The ending inventory was 25% complete as to the conversion cost. At the beginning of the process, 100% of direct material was added. What was the total cost transferred out? A.$130,500 B.$126,973 C.$117,450 D.$121,500

D.$121,500Answer (D) is correct. Total EUP for materials equal 15,000 because 100% of materials are added at the beginning of the process. The materials cost per EUP is $5 ($75,000 materials cost ÷ 15,000 equivalent units). Total EUP for conversion costs equal 13,875 [13,500 physical units completed + (1,500 physical units in ending inventory × 25%)]. The conversion cost is $4 per unit ($55,500 conversion cost ÷ 13,875 equivalent units). Thus, the total cost transferred out is $121,500 [13,500 physical units × ($5 + $4)].

A company's target gross margin is 40% of the selling price of a product that costs $89 per unit. The product's selling price should be A.$142.40 B.$222.50 C.$124.60 D.$148.33

D.$148.33Answer (D) is correct. The gross margin is calculated as [1 - (Unit cost ÷ Unit selling price)]. The question gives a company's target gross margin as 40%. Rearranging the gross margin equation, the unit selling price equals [Unit cost ÷ (1 - Gross margin)]. Thus, the product's selling price is $148.33 [$89 ÷ (1 - .40)].

Rodder, Inc., manufactures a component in a router assembly. The selling price and unit cost data for the component are as follows: Selling price$15Direct materials cost3Direct labor cost3Variable overhead cost3Fixed manufacturing overhead cost2Fixed selling and administration cost1 The company received a special one-time order for 1,000 components. Rodder has an alternative use for production capacity for the 1,000 components that would produce a contribution margin of $5,000. What amount is the lowest unit price Rodder should accept for the component? A.$24 B.$9 C.$12 D.$14

D.$14Answer (D) is correct. The entity should apply relevant costing and contribution margin (revenue - variable cost) analysis. It presumably has two uses for the production capacity needed to make 1,000 components: (1) the special order or (2) an alternative with a unit contribution margin (UCM) of $5 ($5,000 CM ÷ 1,000 units). The fixed costs are not relevant to the choice between the two uses because they will be incurred in either case. Thus, the relevant cost is the unit variable cost ($3 DM + $3 DL + $3 VOH = $9). The unit price at which the entity will be indifferent between the two uses is therefore $14 ($9 unit variable cost + $5 UCM for the alternative to the special order).

Albany Mining Corporation uses a process costing system for its ore extraction operations. The following information pertains to work-in-process inventories and operations for the month of May: Completion % Units Materials Conversion WIP on May 1 32,000 60% 20% Started in production 200,000 Completed production 184,000 WIP on May 31 48,000 90% 40% Costs for the month were as follows: BWIP Incurred in May Materials $54,560 $ 468,000 Direct labor 20,320 182,880 Manufacturing overhead 15,240 391,160 $90,120 $1,042,040 Using the FIFO method, Albany Mining's cost per equivalent unit for conversion costs is A.$3.00 B.$3.10 C.$3.23 D.$2.92

D.$2.92Answer (D) is correct. Under the FIFO method, EUP are determined based only on work performed during the current period. Thus, units in beginning work-in-process must be excluded.ConversionUnits transferred out184,000Add: EWIP (48,000 × 40%)19,200Total completed units203,200Minus: BWIP (32,000 × 20%)(6,400)EUP196,800Total conversion costs incurred for the month of $574,040 ($182,880 DL + $391,160 MOH) spread over 196,800 EUP results in a per-unit cost of $2.917.

The following information is available on MCH Co.'s two product lines:ApplesOrangesSales$ 200,000$ 90,000Variable costs(130,000)(70,000)Contribution margin$ 70,000$ 20,000Fixed costs(30,000)(35,000)Operating income (loss)$ 40,000$(15,000)Assume MCH could discontinue its orange line and rent a factory. At what amount of annual rental income is MCH indifferent between producing oranges and renting a factory space? A.$35,000 B.$55,000 C.$15,000 D.$20,000

D.$20,000Answer (D) is correct. When making a decision, the only relevant revenues and costs are those that will change depending on which choice is made. The entity's current operating income is $25,000 ($40,000 - $15,000). If orange production is discontinued, fixed costs are not affected, and rental of the factory space equals $20,000, operating income also will be $25,000 ($5,000 + $20,000).

A company uses a process costing system to record inventory costs. Data at the end of the month are as follows:UnitsDirect materials (% complete)Conversion cost (% complete)Work-in-process inventory100100%80%Finished goods inventory900100%100%The company uses the weighted-average method to compute the costs. Its total costs for the month are $10,000 for direct materials and $19,600 for conversion costs. What is the total conversion cost per equivalent unit? A.$30.00 B.$24.50 C.$19.60 D.$20.00

D.$20.00Answer (D) is correct. Under the weighted-average method, units in beginning work-in-process (WIP) are treated the same as units started and completed during the current period. However, in this question, the company has no beginning WIP. EUP are calculated by the sum of total units completed and transferred out in the current period plus the proportion of completed ending WIP (ending WIP × percent completed). Thus, EUP of conversion costs is 980 [900 + (100 × 80%)]. The cost per EUP under the weighted-average method is calculated as beginning WIP costs plus current-period costs divided by EUP. Because conversion costs totaled $19,600 for the month, and no beginning WIP existed, conversion costs per EUP is $20 ($19,600 current period costs ÷ 980 EUP).

Product Cott has sales of $200,000, a contribution margin of 20%, and a margin of safety of $80,000. What is Cott's fixed cost? A.$96,000 B.$80,000 C.$16,000 D.$24,000

D.$24,000Answer (D) is correct. Sales minus the margin of safety equals the breakeven point ($200,000 - $80,000 = $120,000). Fixed costs equal the contribution margin at the breakeven point, so fixed costs are $24,000 ($120,000 × 20%).

Mili Co. plans to discontinue a division with a $20,000 contribution to overhead. Overhead allocated to the division is $50,000, of which $5,000 cannot be eliminated. The effect of this discontinuance on Mili's pretax income would be an increase of A.$5,000 B.$30,000 C.$20,000 D.$25,000

D.$25,000Answer (D) is correct. This disinvestment decision eliminates $45,000 of overhead ($50,000 - $5,000) and the $20,000 contribution to overhead. The net effect on pretax income is therefore a $25,000 increase ($45,000 - $20,000).

Pelder Products Company manufactures two types of engineering diagnostic equipment used in construction. The two products are based on different technologies, X-ray and ultrasound, but are manufactured in the same factory. Pelder has computed the manufacturing cost of the X-ray and ultrasound products by adding together direct materials, direct labor, and overhead cost applied based on the number of direct labor hours. The factory has three overhead departments that support the single production line that makes both products. Budgeted overhead spending for the departments is as follows: DepartmentEngineering designMaterial handlingSetupTotal$6,000$5,000$3,000$14,000Pelder's budgeted manufacturing activities and costs for the period are as follows: ProductActivityX-RayUltrasoundUnits produced and sold50100Direct materials used$5,000$8,000Direct labor hours used100300Direct labor cost$4,000$12,000Number of parts used400600 Number of engineering changes 2 1 Number of product setups 8 7 The budgeted cost to manufacture one ultrasound machine using the activity-based costing method is A.$305 B.$293 C.$225 D.$264

D.$264Answer (D) is correct. Charges for direct materials and direct labor are traceable to each type of machine ($8,000 and $12,000 respectively for the ultrasound). The departmental costs must be allocated based on each machine's proportional driver level. Engineering design costs can be allocated to the ultrasound machine at a rate of 33.3% [1 ÷ (1 + 2)], material handling at a rate of 60% [600 ÷ (600 + 400)], and setup at a rate of 46.7% [7 ÷ (7 + 8)]. Pelder's cost for a single ultrasound machine can thus be calculated as follows: Direct materials ($8,000)$ 80Direct labor ($12,000)120Engineering changes ($6,000 × 33.3%) 20Materials handling ($5,000 × 60%) 30Setup ($3,000 × 46.7%)14Total$264

Milo Corp. sells washers for $350 per unit and incurs product costs of $300. Prime costs for the washers are $150 and the fixed portion of the overhead is $90. Milo receives a special order for 2,000 washers from Adobe. Additional variable manufacturing costs to meet Adobe specifications are $40 per unit. Milo has sufficient excess capacity and has determined that this sale will not interfere with regular business. What price must Milo charge Adobe to obtain a 20% markup on variable manufacturing costs? A.$210 B.$252 C.$250 D.$300

D.$300Answer (D) is correct. Prime cost is defined as costs directly attributable to a product. It is calculated as direct materials plus direct labor. Manufacturing overhead consists of all costs of manufacturing that are not direct materials or direct labor. Because Milo has product costs of $300, prime costs of $150, and fixed overhead of $90, it has variable overhead of $60 ($300 - $150 - $90). Moreover, because prime costs are variable and additional variable manufacturing costs for the special order are $40, Milo has total variable manufacturing costs of $250 ($150 prime costs + $60 variable overhead + $40 additional variable manufacturing costs). Thus, Milo must charge $300 [$250 + ($250 × 20%)] to obtain a 20% markup on variable manufacturing costs.

Regan Company operates its factory on a two-shift basis and pays a late-shift differential of 15%. Regan also pays a premium of 50% for overtime work. Because Regan manufactures only for stock, the cost system provides for uniform direct-labor hourly charges for production done without regard to shift worked or work done on an overtime basis. Overtime and late-shift differentials are included in Regan's factory overhead application rate. The May payroll for production workers is as follows: Wages at base direct-labor rates$325,000Shift differentials25,000Overtime premiums10,000 For the month of May, what amount of direct labor should Regan charge to work-in-process? A.$360,000 B.$350,000 C.$335,000 D.$325,000

D.$325,000Answer (D) is correct. Regan's cost system provides for uniform direct hourly charges for production done without regard to shift work or work done on an overtime basis. The shift pay differentials and overtime premiums are included in factory overhead. Accordingly, both the $25,000 and $10,000 amounts should be charged to overhead, and $325,000 should be charged to the WIP account as direct labor.

DJ Co. has a job-order cost system. The following debits (credits) appeared in the work-in-process account for the month of March:March 1, balance$ 12,000March 31, direct materials40,000March 31, direct labor30,000March 31, manufacturing overhead applied27,000March 31, to finished goods(100,000)DJ Co. applies overhead at a predetermined rate of 90% of direct labor cost. Job No. 101, the only job still in process at the end of March, has been charged with manufacturing overhead of $2,250. What was the amount of direct materials charged to Job No. 101? A.$2,500 B.$4,725 C.$2,250 D.$4,250

D.$4,250Answer (D) is correct. The amount DJ charged to direct materials can be calculated as follows:March 1, balance$ 12,000March 31, direct materials40,000March 31, direct labor30,000March 31, manufacturing overhead applied27,000Total debits to work-in-process$ 109,000March 31, to finished goods(100,000)Debit balance in work-in-process (all Job No. 101)$ 9,000Overhead charged to job(2,250)Direct labor charged to job ($2,250 ÷ 90%)*(2,500)Direct materials charged to job$ 4,250*The direct labor cost is multiplied by 90% in determining the total overhead applied to all jobs to calculate the direct labor cost associated with the project; the overhead is divided by 90%.

Pinecrest Co. had variable costs of 25% of sales and fixed costs of $30,000. Pinecrest's breakeven point in sales dollars was A.$24,000 B.$120,000 C.$30,000 D.$40,000

D.$40,000Answer (D) is correct. The breakeven point in dollars equals fixed costs ($30,000) divided by the contribution margin ratio (1.0 - .25), or $40,000.

The following information is taken from Wampler Co.'s current-year contribution income statement: Sales$200,000Contribution margin120,000Fixed costs90,000Income taxes12,000 What was Wampler's margin of safety? A.$182,000 B.$150,000 C.$168,000 D.$50,000

D.$50,000Answer (D) is correct. The margin of safety is the excess of sales over breakeven sales. Thus, income taxes are not relevant because the margin of safety is a pretax amount. Sales are given ($200,000). Breakeven sales in dollars can be calculated as follows: Breakeven sales=Fixed costs ÷ Contr. margin ratio=$90,000 ÷ ($120,000 ÷ $200,000)=$90,000 ÷ 0.6=$150,000The margin of safety is thus $50,000 ($200,000 sales - $150,000 BE sales).

A sporting goods manufacturer buys wood as a direct material for baseball bats. The Forming Department processes the baseball bats, and the bats are then transferred to the Finishing Department, where a sealant is applied. The Forming Department began manufacturing 10,000 "Casey Sluggers" during the month of May. There was no beginning inventory. Costs for the Forming Department for the month of May were as follows: Direct materials $33,000 Conversion costs 17,000 Total $50,000 A total of 8,000 bats were completed and transferred to the Finishing Department; the remaining 2,000 bats were still in the forming process at the end of the month. All of the Forming Department's direct materials were placed in process, but, on average, only 25% of the conversion cost was applied to the ending work-in-process inventory. The cost of the work-in-process inventory in the Forming Department at the end of May is A.$20,000 B.$2,500 C.$10,000 D.$7,600

D.$7,600Answer (D) is correct. The EUP for materials equal 10,000 (8,000 + 2,000) because the work-in-process is 100% complete as to materials. Thus, dividing the $33,000 by 10,000 units results in a unit cost for materials of $3.30. The EUP for conversion costs equal 8,500 units [8,000 + (2,000 units × .25)]. Dividing the $17,000 of conversion costs by 8,500 EUP results in a unit cost of $2 per bat, and the total cost of goods transferred out is $5.30, consisting of $3.30 for materials and $2 for conversion costs. Multiplying $5.30 by the 8,000 bats completed results in a total transfer of $42,400. Consequently, the cost of the ending work-in-process must have been $7,600 ($50,000 total costs incurred - $42,400).

The following information for a company is given:Fixed cost per month$2,500Unit selling price$ 100Variable cost as a percentage of sales60%What amount of annual sales must the company achieve to break even? A.$50,000 B.$100,000 C.$30,000 D.$75,000

D.$75,000Answer (D) is correct. The breakeven point in sales dollars equals fixed costs divided by the contribution margin ratio (CMR). CMR is the ratio of contribution margin to sales price on either a total or per-unit basis. The total annual fixed cost is $30,000. The contribution margin per unit is $40 ($100 selling price - $60 variable costs). The CMR equals 0.4 ($40 contribution margin ÷ $100 selling price). Thus, the company's breakeven point in annual sales dollars equals $75,000 ($30,000 annual fixed costs ÷ 0.4 CMR).

Worley Company has underapplied overhead of $45,000 for the year. Before disposition of the underapplied overhead, selected year-end balances from Worley's accounting records were Sales$1,200,000Cost of goods sold720,000Direct materials inventory36,000Work-in-process inventory54,000Finished goods inventory90,000 Under Worley's cost accounting system, over- or underapplied overhead is assigned to appropriate inventories and COGS based on year-end balances. In its year-end income statement, Worley should report COGS of A.$765,000 B.$682,500 C.$684,000 D.$757,500

D.$757,500Answer (D) is correct. The assignment of underapplied overhead increases COGS. The underapplied overhead of $45,000 for the year should be assigned on a pro rata basis to work-in-process ($54,000), finished goods ($90,000), and COGS ($720,000). The sum of these three items is $864,000. Thus, $37,500 should be assigned to COGS [($720,000 ÷ $864,000) × $45,000]. COGS after assignment is $757,500 ($37,500 + $720,000). The remaining $7,500 should be assigned proportionately to work-in-process and finished goods.

Hamilton Company uses job-order costing. Manufacturing overhead is applied to production at a predetermined rate of 150% of direct labor cost. Any over- or underapplied overhead is closed to the cost of goods sold account at the end of each month. Additional information is available as follows: Job 101 was the only job in process at January 31, with accumulated costs as follows:Direct materials$4,000Direct labor2,000Applied manufacturing overhead3,000Total manufacturing costs$9,000 Jobs 102, 103, and 104 were started during February. Direct materials requisitions for February totaled $26,000. Direct labor cost of $20,000 was incurred for February. Actual manufacturing overhead was $32,000 for February. The only job still in process on February 28 was Job 104, with costs of $2,800 for direct materials and $1,800 for direct labor. The cost of goods manufactured for February was A.$79,700 B.$85,000 C.$78,000 D.$77,700

D.$77,700Answer (D) is correct. COGM is the sum of the costs in BWIP and all the costs incurred during the period minus the costs in EWIP. The calculation of COGM uses applied overhead ($30,000 = $20,000 DL cost × 150%). The $7,300 in EWIP includes $2,800 for direct materials, $1,800 for direct labor, and $2,700 for applied overhead (at 150% of DL cost).BWIP$ 9,000 Direct labor20,000 Applied overhead 30,000 Direct materials26,000 EWIP(7,300)COGM$77,700

S Corp. produces premium office chairs. Due to a recent change in market share, S must decide whether to make or buy an order of 1,000 chairs. The materials cost of producing a chair is $20 per pound, the cost of direct labor is $40 per direct labor hour, and manufacturing overhead (100% variable) is allocated at a rate of $10 per chair. Fixed costs for the year are $5 per chair, of which 20% are avoidable. Each chair requires 2 pounds of materials and 1 hour of direct labor to complete. Assuming that S has excess capacity, the total cost per chair relevant to the make-or-buy decision is A.$90 B.$95 C.$71 D.$91

D.$91Answer (D) is correct. Given sufficient available capacity, no opportunity cost exists. Relevant costs are limited to variable costs and avoidable fixed costs. Thus, on a per-unit basis, each chair has a relevant cost of $91 [(2 lbs. of material × $20) + (1 direct labor hour × $40) + $10 variable manufacturing overhead + ($5 fixed costs × 20% avoidable)].

The data below pertain to two types of products manufactured by Cobb Corp. Fixed costs total $300,000 annually. The expected mix in units is 60% for product Y and 40% for product Z. Per Unit Sales Price Variable Costs Product Y $120 $ 70 Product Z 500 200 How much is Cobb's breakeven point in units? A.2,459 B.1,111 C.857 D.2,000

D.2,000Answer (D) is correct. The BEP in units is equal to fixed costs divided by the unit contribution margin (UCM). The weighted-average UCM is $150, calculated as follows:Product YProduct ZSales price$120$500Minus: variable costs(70)(200)Contribution margin$ 50$300Times: mix ratio× 60%× 40%Weighted contribution margin$ 30$120The BEP is 2,000 units ($300,000 fixed costs ÷ $150 UCM).

State College is using cost-volume-profit analysis to determine tuition rates for the upcoming school year. Projected costs for the year are as follows:Contribution margin per student$ 1,800Variable expenses per student1,000Total fixed expenses360,000Based on these estimates, what is the approximate breakeven point in number of students? A.450 B.129 C.360 D.200

D.200Answer (D) is correct. The breakeven point in units equals fixed costs divided by unit contribution margin. Thus, the approximate breakeven point in number of students is 200 ($360,000 fixed expenses ÷ $1,800 contribution margin per student).

Green Co. produces only Product Z. As part of the annual budgeting, Green is considering whether to produce a new product. Green's CFO obtained information from various departments within the company. The plant manager expected the following costs would be incurred in producing the new product: Direct materials$1 per unitDirect labor$100 per hourFixed cost$55,000The marketing manager decided to spend $2 per unit for the first 5,000 items sold with no additional costs after that. The marketing manager confirmed that the current market price for the new product was $4,000 per 1,000 units. The plant manager told the CFO that the employees would be able to produce 500 units per hour. Approximately how many units would Green have to sell to break even? A.19,643 B.68,750 C.54,167 D.23,215

D.23,215Answer (D) is correct. The breakeven point in units is calculated as follows: Fixed costs ÷ Unit contribution margin. Unit contribution margin equals selling price per unit minus variable costs per unit. Accordingly, Green's breakeven point in units is 23,215, as calculated below:Explicit fixed cost$55,000Marketing costs10,000($2 × 5,000 units)Total fixed cost$65,000Selling price per unit$4.00($4,000 ÷ 1,000 units)Direct materials per unit(1.00)Direct labor per unit(.20)($100 per hr. ÷ 500 units per hr.)Unit contribution margin$2.80Breakeven units equal 23,215 ($65,000 total fixed costs ÷ $2.80 unit contribution margin).

The following information pertains to Sisk Co.:Sales (25,000 units)$500,000Direct materials and direct labor150,000Factory overhead:Variable20,000Fixed35,000Selling and general expenses:Variable5,000Fixed30,000 Sisk's breakeven point in number of units is A.6,250 B.4,924 C.9,286 D.5,000

D.5,000Answer (D) is correct. The breakeven point in units equals the fixed costs divided by the unit contribution margin (UCM). The fixed costs are $65,000 ($35,000 manufacturing overhead + $30,000 SG&A). The UCM is calculated as follows: DollarsUnitsPer UnitSales$500,000÷25,000=$20.00Variable costs:Prime costs$150,000Variable overhead20,000Variable SG&A5,000Total var. costs$175,000÷25,000=(7.00)Contribution margin$13.00Thus, the breakeven point in units is 5,000 ($65,000 fixed costs ÷ $13 UCM).

A company uses weighted-average process costing for the product it manufactures. All direct materials are added at the beginning of production, and conversion costs are applied evenly during production. The following data apply to the past month:Total units in beginning inventory(30% complete as to conversion costs)1,500Total units transferred to finished goods inventory7,400Total units in ending inventory(60% complete as to conversion costs)2,300 Assuming no spoilage, equivalent units of production (EUP) with respect to conversion costs total A.8,330 B.9,230 C.9,700 D.8,780

D.8,780Answer (D) is correct. The weighted-average method averages the work performed in the prior period with the work done in the current period. Thus, beginning work-in-process is left in the calculation (unlike FIFO). ConversionUnits transferred out7,400Add: EWIP (2,300 × 60%)1,380EUP8,780

Under a job-order system of cost accounting, the dollar amount of the general ledger entry involved in the transfer of inventory from work-in-process to finished goods is the sum of the costs charged to all jobs A.Completed and sold during the period. B.Started in process during the period. C.In process during the period. D.Completed during the period.

D.Completed during the period.Answer (D) is correct. The entry to transfer inventory from WIP to FG is to debit finished goods and credit work-in-process. The amount of the entry is the sum of the costs (irrespective of the period in which they were incurred) charged to all jobs completed during the period.

The most likely strategy to reduce the breakeven point would be to A.Decrease both the fixed costs and the contribution margin. B.Increase both the fixed costs and the contribution margin. C.Increase the fixed costs and decrease the contribution margin. D.Decrease the fixed costs and increase the contribution margin.

D.Decrease the fixed costs and increase the contribution margin.Answer (D) is correct. A ratio can be reduced either by decreasing the numerator or increasing the denominator. The breakeven point in units equals fixed costs divided by the unit contribution margin. The breakeven point in sales dollars is the fixed costs divided by the contribution margin ratio. Because fixed costs are in the numerator and the contribution margin is in the denominator, decreasing the fixed costs and increasing the contribution margin reduces the breakeven point.

Richardson Motors uses 10 units of Part No. T305 each month in the production of large diesel engines. The cost to manufacture one unit of T305 is presented as follows: Direct materials $ 2,000 Materials handling (20% of direct materials cost) 400 Direct labor 16,000 Manufacturing overhead (150% of direct labor) 24,000 Total manufacturing cost $42,400 Materials handling, which is not included in manufacturing overhead, represents the direct variable costs of the receiving department that are applied to direct materials and purchased components on the basis of their cost. Richardson's annual manufacturing overhead budget is one-third variable and two-thirds fixed. Simpson Castings, one of Richardson's reliable vendors, has offered to supply T305 at a unit price of $30,000. If Richardson Motors purchases the ten T305 units from Simpson Castings, the capacity Richardson used to manufacture these parts would be idle. Should Richardson decide to purchase the parts from Simpson, the out-of-pocket cost per unit of T305 would A.Decrease $6,400. B.Decrease $12,400. C.Increase $3,600. D.Increase $9,600.

D.Increase $9,600.Answer (D) is correct. The out-of-pocket cost of making the part equals the total manufacturing cost minus the fixed overhead, or $26,400 {$42,400 - [(2 ÷ 3) × $24,000]}. The cost of the component consists of the $30,000 purchase price plus the $6,000 (20% of cost) of variable receiving costs, or a total of $36,000. Thus, unit out-of-pocket cost would increase by $9,600 if the components were purchased.

Activity-based costing (ABC) is a term used to describe allocation methods that are based on cause and effect (i.e., charging costs to whatever causes them). Why is this more important to project managers using job-order costing than those using process costing? A.If a project manager is charged for costs used, such as equipment rental, resources tend to sit idle in case they are needed. B.If project managers are charged an in-house rate for resources that exceeds the rate charged by an external supplier, they will not use company resources that sit idle. Total costs then decrease. C.Process costing is no longer used. D.Job-order costing is used if the products or services sold are different enough to justify the additional cost involved in tracing costs to their cause. Charging all products or services a flat amount per unit shifts costs from those that cause them to those that do not.

D.Job-order costing is used if the products or services sold are different enough to justify the additional cost involved in tracing costs to their cause. Charging all products or services a flat amount per unit shifts costs from those that cause them to those that do not.Answer (D) is correct. The theory is to hold an individual accountable only for those costs (s)he controls.

Which costing method is a custom home builder most likely to use? A.Process costing. B.Process costing to a split-off point and job-order costing afterwards. C.None of the answers are correct. D.Job-order costing.

D.Job-order costing.Answer (D) is correct. Job-order costing is used when each end product is unique. Process cost accounting assigns costs to inventoriable goods or services. It applies to relatively homogeneous products that are mass produced on a continuous basis (e.g., petroleum products, thread, and computer monitors). The end products made by a custom home builder are customized and unique. Thus, a custom home builder is most likely to use job-order costing.

Regis Company manufactures plugs used in its manufacturing cycle at a cost of $36 per unit that includes $8 of fixed overhead. Regis needs 30,000 of these plugs annually, and Orlan Company has offered to sell these units to Regis at $33 per unit. If Regis decides to purchase the plugs, $60,000 of the annual fixed overhead applied will be eliminated, and the company may be able to rent the facility previously used for manufacturing the plugs. If Regis Company purchases the plugs but does not rent the unused facility, the company would A.Lose $6.00 per unit. B.Save $3.00 per unit. C.Save $2.00 per unit. D.Lose $3.00 per unit.

D.Lose $3.00 per unit.Answer (D) is correct. Exclusive of the fixed overhead, the unit cost of making the plugs is $28 ($36 total cost - $8 fixed OH). Purchasing the plugs will avoid $2 per unit of fixed overhead ($60,000 OH applied ÷ 30,000 units). Accordingly, $6 per unit of fixed overhead is unavoidable, and the relevant (avoidable) unit cost of making the plugs is $30 [$36 total cost - ($8 fixed OH - $2 avoidable cost)]. The purchase option therefore results in a $3-per-unit loss ($33 purchase price - $30 relevant cost).

A company has received an offer from a supplier to produce units that the company currently produces and sells. The unit price quoted by the supplier is higher than the company's variable production cost per unit but lower than the price at which the company can market the units. Under which circumstance would the company's profits increase by purchasing units from the supplier? A.The company has significant sunk costs. B.The company's administrative costs are zero. C.The company's fixed overhead would remain the same if the company purchased units from the supplier. D.Market demand for the product exceeds the company's capacity.

D.Market demand for the product exceeds the company's capacity. Answer (D) is correct. Fixed costs are relevant costs in the absence of idle capacity. Thus, when demand for a product exceeds a company's capacity to produce a product, fixed costs may be avoided by purchasing the product from a supplier, resulting in a decrease in expenses and an increase in profits.

Weighted-average and first-in, first-out (FIFO) equivalent units would be the same in a period when which of the following occurs? A.No ending inventory exists. B.Both a beginning and an ending inventory exist but are not necessarily equal. C.Beginning inventory units equal ending inventory units. D.No beginning inventory exists.

D.No beginning inventory exists. Answer (D) is correct. Equivalent units of production (EUP) are the number of complete goods that could have been produced using the inputs consumed during the period. Two methods of calculating EUP are in common use: weighted-average and FIFO. Under the weighted-average method, units in beginning inventory are treated as if they were started and completed during the current period. Beginning inventory is therefore not included in the EUP calculation. Under the FIFO method, units in beginning inventory are part of the EUP calculation. If no beginning inventory exists, then weighted-average and FIFO EUP would be the same.

Stewart Industries has been producing two bearings, components B12 and B18, for use in production. B12 B18 Machine hours required per unit 2.5 3.0 Standard cost per unit: Direct material $ 2.25 $ 3.75 Direct labor 4.00 4.50 Manufacturing overhead: Variable (See Note 1) 2.00 2.25 Fixed (See Note 2) 3.75 4.50 $12.00 $15.00 Stewart's annual requirement for these components is 8,000 units of B12 and 11,000 units of B18. Recently, Stewart's management decided to devote additional machine time to other product lines resulting in only 41,000 machine hours per year that can be dedicated to the production of the bearings. An outside company has offered to sell Stewart the annual supply of the bearings at prices of $11.25 for B12 and $13.50 for B18. Stewart wants to schedule the otherwise idle 41,000 machine hours to produce bearings so that the company can minimize its costs (maximize its net benefits). Note 1: Variable manufacturing overhead is applied on the basis of direct labor hours. Note 2: Fixed manufacturing overhead is applied on the basis of machine hours. Stewart will maximize its net benefits by A.Purchasing 11,000 units of B18 and manufacturing 8,000 units of B12. B.Purchasing 8,000 units of B12 and manufacturing 11,000 units of B18. C.Purchasing 4,800 units of B12 and manufacturing the remaining bearings. D.Purchasing 4,000 units of B18 and manufacturing the remaining bearings.

D.Purchasing 4,000 units of B18 and manufacturing the remaining bearings.Answer (D) is correct. Purchasing will increase the company's costs by $3 ($11.25 - $2.25 - $4 - $2) for each B12 bearing, or $1.20 per hour ($3 ÷ 2.5 hrs). Buying B18 will only cost the company an additional $1 per machine hour [($13.50 - $3.75 - $4.50 - $2.25) ÷ 3 machine hours]. Thus, the company should make all the needed B12s and compensate for the machine hours constraint by purchasing B18s. Given that each unit of B12 requires 2.5 hours of machine time, the company can produce the needed 8,000 units in 20,000 hours (2.5 × 8,000). The remaining 21,000 hours (41,000 - 20,000) can then be used for the production of 7,000 B18s (21,000 ÷ 3 hrs.). Because the annual requirement of B18s is 11,000 units, the other 4,000 units will have to be purchased.

Nile Co. is a manufacturer whose cost assignment and product costing procedures follow activity-based costing principles. Activities have been identified and classified as being either value-adding or nonvalue-adding as to each product. Which of the following activities used in Nile's production process is nonvalue-adding? A.Heat treatment activity. B.Drill press activity. C.Design engineering activity. D.Raw materials storage activity.

D.Raw materials storage activity.Answer (D) is correct. Analysis by activity provides for better cost control because of identification of nonvalue-adding activities. A value-added activity contributes to customer satisfaction or meets a need of the entity. A nonvalue-adding activity does not make such a contribution. It can be eliminated, reduced, or redesigned without impairing the quantity, quality, or responsiveness of the product or service desired by customers or the entity. For example, raw materials storage may be greatly reduced or eliminated in a just-in-time (JIT) production system without affecting customer value.

When using activity-based costing techniques, which one of the following departmental activities would be expected to use machine hours as a cost driver to allocate overhead costs to production? A.Machine setups. B.Material handling. C.Plant cafeteria. D.Robotics painting.

D.Robotics painting.Answer (D) is correct. Machine hours are a direct measure of the level of use of a robotic painting operation.

Breakeven analysis assumes over the relevant range that A.Variable costs are nonlinear. B.Fixed costs are nonlinear. C.Selling prices are nonlinear. D.Total costs are linear.

D.Total costs are linear.Answer (D) is correct. Breakeven analysis assumes that the cost and revenue factors used in the formula are linear and do not fluctuate with volume. Hence, fixed costs are deemed to be fixed over the relevant range of volume, and variable cost per unit remains constant as volume changes within the relevant range.

In a job-order cost system, direct labor costs usually are recorded initially as an increase in A.Factory overhead applied. B.Factory overhead control. C.Finished goods control. D.Work-in-process control.

D.Work-in-process control.Answer (D) is correct. Direct labor costs are inventoriable costs. They are initially debited to the work-in-process control account.

In a job-order cost system, the application of factory overhead is usually reflected in the general ledger as an increase in A.Factory overhead control. B.Cost of goods sold. C.Finished goods control. D.Work-in-process control.

D.Work-in-process control.Answer (D) is correct. The entry to record the application of factory overhead to specific jobs is to charge WIP control and credit factory overhead applied using a predetermined overhead rate. The effect is to increase the WIP control account.

In a job-order cost system, the use of direct materials previously purchased usually is recorded as an increase in A.Factory overhead applied. B.Stores control. C.Factory overhead control. D.Work-in-process control.

D.Work-in-process control.Answer (D) is correct. The purchase of direct materials requires a debit to (an increase in) direct materials inventory (stores control). This account is credited and work-in-process control is debited when direct materials are issued to a production department.

In a process cost system, the application of factory overhead usually is recorded as an increase in A.Factory overhead control. B.Finished goods inventory control. C.Cost of goods sold. D.Work-in-process inventory control.

D.Work-in-process inventory control.Answer (D) is correct. The principal distinction between process costing and job-order costing systems is that the latter use subsidiary WIP and finished goods ledgers to account for separate jobs. However, the same general ledger accounts are used in both systems, and cost flow among accounts is also the same. Both systems increase work-in-process control to record applied overhead.

A company is considering the implementation of an activity-based costing and management program. The company A.Would likely use fewer cost pools than it did under more traditional accounting methods. B.Would probably find a lack of software in the marketplace to assist with the related recordkeeping. C.Should focus on manufacturing activities and avoid implementation with service-type functions. D.Would normally gain added insights into causes of cost.

D.Would normally gain added insights into causes of cost.Answer (D) is correct. One of the benefits of activity-based costing is the discovery of cost relationships that are unnoticed using traditional accounting methods.


Set pelajaran terkait

Module 2 - Unit 1: Digital Content

View Set

NR599 Nursing Informatics: Mid Term_2021

View Set

Chapter 2: Reading/Study Guide Terms Review

View Set